Med-Surg 1 - Final Review

Ace your homework & exams now with Quizwiz!

A nurse evaluates the following laboratory results for a patient who has hypoparathyroidism: Calcium 7.2 mg/dL (1.8 mmol/L)Sodium 144 mEq/L (144 mmol/L)Magnesium 1.2 mEq/L (0.6 mmol/L)Potassium 5.7 mEq/L (5.7 mmol/L)Based on these results, which medications does the nurse anticipate administering? (Select all that apply.) a. 50% magnesium sulfate b. Oral potassium chloride c. Oral calcitriol (Rocaltrol) d. Intravenous calcium chloride e. 3% normal saline IV solution

A. 50% magnesium sulfate D. Intravenous calcium chloride

A patient with the diagnosis of chronic obstructive pulmonary disease (COPD) becomes increasingly short of breath. Which nursing intervention is most appropriate? a. Administer O2 using a Venturi mask at 24% b. Use nasal cannula to administer high flow oxygen c. Do not administer O2 due to history of COPD d. Begin oxygen therapy using a simple face mask at 8L

A. Administer O2 using a Venturi mask at 24%

While assessing a patient on a cardiac unit, a nurse identifies the presence of an S3 gallop (extra heart sound). What action would the nurse take next? a. Assess for symptoms of left-sided heart failure. b. Transfer the patient to the intensive care unit. c. Document this as a normal finding. d. Call the healthcare provider immediately.

A. Assess for symptoms of left-sided heart failure.

A nurse assesses a patient's electrocardiogram (ECG) and observes the reading. How would the nurse document this patient's ECG strip? a. Sinus rhythm with premature ventricular contractions (PVCs) b. Ventricular tachycardia c. Sinus rhythm with premature atrial contractions (PACs) d. Ventricular fibrillation

A. Sinus rhythm with premature ventricular contractions (PVCs)

A nurse assesses a patient with atrial fibrillation. Which manifestation would alert the nurse to the possibility of a serious complication from this condition? a. Speech alterations b. Sinus tachycardia c. Fatigue d. Dyspnea with activity

A. Speech alterations

The nurse is reviewing the lipid panel of a male patient who has atherosclerosis. Which finding is most concerning? a. Triglycerides: 198 mg/dL b. Cholesterol: 126 mg/dL c. High-density lipoprotein cholesterol (HDL-C): 48 mg/dL d. Low-density lipoprotein cholesterol (LDL-C): 122 mg/dL

A. Triglycerides: 198 mg/dL

A nurse cares for a patient who is prescribed vasopressin (DDAVP) for diabetes insipidus. Which assessment findings indicate a therapeutic response to this therapy? (Select all that apply.) a. Urine output is decreased. b. Urine osmolality is decreased. c. Specific gravity is increased. d. Urine osmolality is increased. e. Specific gravity is decreased. f. Urine output is increased.

A. Urine output is decreased C. Specific gravity is increased D. Urine osmolality is increased.

A client who has had a myocardial infarction asks the nurse why she should not bear down or strain to ensure having a bowel movement. The nurse provides education to the client based on which physiological concept? a. Vagus nerve stimulation causes a decrease in heart rate and cardiac contractility. b. Sympathetic nerve stimulation causes a decrease in heart rate and cardiac contractility. c. Sympathetic nerve stimulation causes an increase in heart rate and cardiac contractility. d. Vagus nerve stimulation causes an increase in heart rate and cardiac contractility.

A. Vagus nerve stimulation causes a decrease in HR and cardiac contractility Rationale: Bearing down as if straining to have a bowel movement can stimulate a vagal reflex. Stimulation of the vagus nerve causes a decrease in heart rate and cardiac contractility. Stimulation of the sympathetic nervous system has the opposite effect. These two branches of the autonomic nervous system oppose each other to maintain homeostasis.

The nurse is assessing a client with a suspected diagnosis of hypocalcemia. Which clinical manifestation would the nurse expect to note in the client? a. Twitching b. Hypoactive bowel sounds c. Negative Trousseau's sign d. Hypoactive deep tendon reflexes

ANS: A Rationale: The normal serum calcium level is 9 to 10.5 mg/dL (2.25 to 2.75 mmol/L). A serum calcium level lower than 9 mg/dL (2.25 mmol/L) indicates hypocalcemia. Signs of hypocalcemia include paresthesias followed by numbness, hyperactive deep tendon reflexes, and a positive Trousseau's or Chvostek's sign. Additional signs of hypocalcemia include increased neuromuscular excitability, muscle cramps, twitching, tetany, seizures, irritability, and anxiety. Gastrointestinal symptoms include increased gastric motility, hyperactive bowel sounds, abdominal cramping, and diarrhea. Test-Taking Strategy: Note that the three incorrect options reflect a hypoactivity rather than excitability.

A client is admitted to the ambulatory surgery center for elective surgery. The nurse asks the client whether any food, fluid, or medication was taken today. Which medication, if taken by the client, should indicate to the nurse the need to contact the health care provider? a. A beta-blocker b. An anticoagulant c. An antibiotic d. A calcium channel blocker

ANS: B Rationale: An anticoagulant suppresses coagulation by inhibiting clotting factors. A client admitted for elective surgery should have been instructed to discontinue the anticoagulant 7 to 10 days preoperatively. Even if this were unscheduled surgery, the nurse should notify the health care provider. Vitamin K can be given for reversal of its action, but the client may still have an increased risk of bleeding. The other medications listed are commonly taken and do not constitute an increased risk for the client.

A client has just been admitted to the nursing unit following thyroidectomy. Which assessment is the priority for this client? a. Level of hoarseness b. Respiratory distress c. Hypoglycemia d. Edema at the surgical site

ANS: B Rationale: Thyroidectomy is the removal of the thyroid gland, which is located in the anterior neck. It is very important to monitor airway status, as any swelling to the surgical site could cause respiratory distress. Although all of the options are important for the nurse to monitor, the priority nursing action is to monitor the airway. Test-Taking Strategy: Note the strategic word, priority. Use the ABCs-airway, breathing, and circulation-to assist in directing you to the correct option.

The nurse is monitoring a client newly diagnosed with diabetes mellitus for signs of complications. Which sign or symptom, if exhibited in the client, indicates that the client is at risk for chronic complications of diabetes if the blood glucose is not adequately managed? a. Decreased respiratory rate b. Pedal Edema c. Polyuria d. Diaphoresis

ANS: C Rationale: Chronic hyperglycemia, resulting from poor glycemic control, contributes to the microvascular and macrovascular complications of diabetes mellitus. Classic symptoms of hyperglycemia include polydipsia, polyuria, and polyphagia. Diaphoresis may occur in hypoglycemia. Hypoglycemia is an acute complication of diabetes mellitus; however, it does not predispose a client to the chronic complications of diabetes mellitus. Therefore, option 2 can be eliminated because this finding is characteristic of hypoglycemia. Options 3 and 4 are not associated with diabetes mellitus. Test-Taking Strategy: Focus on the subject, chronic complications of diabetes mellitus. Recall that poor glycemic control contributes to development of the chronic complications of diabetes mellitus. Remember the 3 Ps associated with hyperglycemia—polyuria, polydipsia, and polyphagia.

A nurse cares for a patient with congestive heart failure who has a regular cardiac rhythm of 128 beats/min. For which physiologic alterations would the nurse assess? (Select all that apply.) a. Increase in urine output b. Decrease in cardiac output c. Increase in cardiac output d. Decrease in urine output

B. Decrease in cardiac output D. Decrease in urine output

An emergency department nurse assesses a patient with ketoacidosis. Which clinical manifestation would the nurse correlate with this condition? a. Extremity tremors followed by seizure activity b. Increased rate and depth of respiration c. Oral temperature of 102° F (38.9° C) d. Severe orthostatic hypotension

B. Increased rate and depth of respiration

The nursing student conducting a clinical conference on immunity places an emphasis on active immunity. Which statement by fellow nursing students indicates successful teaching? a. "Active immunity provides protection immediately and forever." b. "Passive immunity can last for years." c. "Active immunity lasts for years and can be easily reactivated by a booster dose of antigen." d. "Active immunity only lasts from days to months."

C. "Active immunity lasts for years and can be easily reactivated by a booster dose of antigen." Rationale: Active immunity lasts for years and is natural by infection or artificial by stimulation of the body's immune defenses for example by vaccination. It can be easily reactivated by a booster dose of antigen. Protection from active immunity takes 5 to 14 days to develop after the first exposure to the antigen and 1 to 3 days after subsequent exposures. Active immunity lasts much longer and is more effective at preventing subsequent infections than passive immunity however it does not last forever. Passively received human antibodies have a half-life of about 30 days. Passive immunity provides protection immediately.

A nurse assesses a patient with pericarditis. Which assessment finding would the nurse expect to find? a. Presence of a regular gallop rhythm b. Coarse crackles in bilateral lung bases c. Friction rub at the left lower sternal border d. Heart rate that speeds up and slows down

C. Friction rub at the left lower sternal border

A patient has been diagnosed with hypertension but does not take the antihypertensive medications because of a lack of symptoms. What response by the nurse is best? a. "You are lucky; most people get severe morning headaches." b. "You need to take your medicine or you will get kidney failure." c. "Do you have trouble affording your medications?" d. "Most people with hypertension do not have symptoms."

D. "Most people with hypertension do not have symptoms."

A patient who had a chest tube placed 8 hours ago and refuses to take deep breaths because of the pain. What action is of highest priority? a. Ambulate the patient in the hallway to promote deep breathing. b. Contact the prescriber and request a STAT chest X-ray c. Auscultate the patient's anterior and posterior lung fields. d. Administer pain medication and encourage the patient to take deep breaths.

D. Administer pain medication and encourage the patient to take deep breaths.

What is the MOST important step a nurse can take to prevent anaphylactic shock in a patient? a. Establishing IV access b. Administering Epinephrine c. Assessing, documenting, and avoiding all the patient allergies d. Administering Corticosteroids

C. Assessing, documenting, and avoiding all the patient allergies Rationale: This is the MOST important and easiest step a nurse can take in preventing anaphylactic shock in a patient.

You're explaining to a group of outpatients about the signs and symptoms that may present with osteoarthritis. Select all the signs and symptoms that may present with this condition: a. Bouchard's Node b. Crepitus c. Fever d. Anemia e. Morning stiffness for less than 30 minutes f. Hard and bony joints g. Herberden's Node h. Soft, tender, warm joints

A, B, E, F, G Rationale: These are common findings found in osteoarthritis. Options C, D, and H are found in rheumatoid arthritis

A nurse cares for a patient recovering from prosthetic valve replacement surgery. The patient asks, "Why will I need to take anticoagulants for the rest of my life?" What is the best response by the nurse? a. "Blood clots form more easily in artificial replacement valves." b. "The vein taken from your leg reduces circulation in the leg." c. "The prosthetic valve places you at greater risk for a heart attack." d. "The surgery left a lot of small clots in your heart and lungs."

A. "Blood clots form more easily in artificial replacement valves."

A patient is at risk for septic shock when a microorganism invades the body. Which microorganism is the MOST common cause of sepsis? a. Fungus b. Virus c. Bacteria d. Parasite

C. Bacteria Rationale: Gram-positive or gram-negative bacteria are the MOST common cause of sepsis.

A nurse teaches a patient who is prescribed an unsealed radioactive isotope. Which statements will the nurse include in this patient's education? (Select all that apply.) a. "Do not share utensils, plates, and cups with anyone else." b. "Wash your clothing separate from others in the household." c. "Take a laxative 2 days after therapy to excrete the radiation." d. "You can play with your grandchildren for 1 hour each day."

A. "Do not share utensils, plates, and cups with anyone else." B. "Wash your clothing separate from others in the household." C. "Take a laxative 2 days after therapy to excrete the radiation."

A nurse assesses a patient on the medical-surgical unit. Which statement made by the patient alerts the nurse to assess the patient for hypothyroidism? a. "I am always tired, even with 12 hours of sleep." b. "I seem to feel the heat more than other people." c. "Food just doesn't taste good without a lot of salt." d. "My sister has thyroid problems."

A. "I am always tired, even with 12 hours of sleep."

A nurse assesses a patient in an outpatient clinic. Which statement alerts the nurse to the possibility of left-sided heart failure? a. "I must stop halfway up the stairs to catch my breath." b. "I am awakened by the need to urinate at night." c. "I have been drinking more water than usual." d. "I have experienced blurred vision on several occasions."

A. "I must stop halfway up the stairs to catch my breath."

After teaching a patient who has an implantable cardioverter-defibrillator (ICD), a nurse assesses the patient's understanding. Which statement by the patient indicates a correct understanding of the teaching? a. "I will avoid sources of strong electromagnetic fields." b. "I should participate in a strenuous exercise program." c. "Now I can discontinue my antidysrhythmic medication." d. "I should wear a snug-fitting shirt over the ICD."

A. "I will avoid sources of strong electromagnetic fields."

The new registered nurse (RN) is orienting on the cardiac unit. Which statement by the new RN indicates an understanding of an early indication of fluid volume deficit due to blood loss? a. "Pulse rate will increase." b. "Crackles in the lungs will be present." c. "Edema will be present in the legs." d. "Blood pressure will decrease."

A. "Pulse rate will increase." Rationale: The cardiac output is determined by the volume of the circulating blood, the pumping action of the heart, and the tone of the vascular bed. Early decreases in fluid volume are compensated for by an increase in the pulse rate. Although the blood pressure will decrease, it is not the earliest indicator. Edema and crackles in the lungs indicate an increase in fluid overload. Test-Taking Strategy: Note the strategic word, early. Eliminate options 3 and 4 first because they both indicate an increase in fluid volume. Regarding the remaining options, think about the physiology of the cardiovascular system and the inherent means of compensation available in that system.

The registered nurse (RN) is educating a new RN about the use of oxygen for clients with angina pectoris. Which statement by the new nurse indicates that the teaching has been effective? a. "The pain of angina pectoris occurs because of a decreased oxygen supply to heart cells." b. "Oxygen will prevent the development of any thrombus." c. "Oxygen has a calming effect." d. "Oxygen dilates the blood vessels so that they can supply more nutrients to the heart muscle."

A. "The pain of angina pectoris occurs because of a decreased oxygen supply to heart cells." Rationale: The pain associated with angina results from ischemia of myocardial cells. The pain often is precipitated by activity that places more oxygen demand on heart muscle. Supplemental oxygen will help meet the added demands on the heart muscle. Oxygen does not dilate blood vessels or prevent thrombus formation and does not directly calm the client. Test-Taking Strategy: Note the strategic word, effective. Focus on the subject, the action of oxygen. Eliminate option 1 because it does not address the physiological necessity of oxygen. Eliminate options 2 and 3 because oxygen does not prevent clot formation or cause vessel dilation.

A nurse cares for a patient who has hypothyroidism as a result of Hashimoto's thyroiditis. The patient asks, "How long will I need to take this thyroid medication?" How does the nurse respond? a. "You'll need thyroid pills for life because your thyroid won't start working again." b. "You will need to take the thyroid medication until the goiter is completely gone." c. "When blood tests indicate normal thyroid function, you can stop the medication." d. "Thyroiditis is cured with antibiotics. Then you won't need thyroid medication."

A. "You'll need thyroid pills for life because your thyroid won't start working again."

The nurse is assessing the functioning of a chest tube drainage system in a client who has just returned from the recovery room following a thoracotomy with wedge resection. Which are the expected assessment findings? Select all that apply. a. 50 mL of drainage in the drainage collection chamber b. Excessive bubbling in the water seal chamber c. Fluctuation of water in the tube in the water seal chamber during inhalation and exhalation d. Drainage system maintained below the client's chest e. Occlusive dressing in place over the chest tube insertion site f. Vigorous bubbling in the suction control chamber

A. 50 mL of drainage in the drainage collection chamber C. Fluctuation of water in the tube in the water seal chamber during inhalation and exhalation D. Drainage system maintained below the client's chest E. Occlusive dressing in place over the chest tube insertion site Rationale: The bubbling of water in the water seal chamber indicates air drainage from the client and usually is seen when intrathoracic pressure is higher than atmospheric pressure, and may occur during exhalation, coughing, or sneezing. Excessive bubbling in the water seal chamber may indicate an air leak, an unexpected finding. Fluctuation of water in the tube in the water seal chamber during inhalation and exhalation is expected. An absence of fluctuation may indicate that the chest tube is obstructed or that the lung has reexpanded and that no more air is leaking into the pleural space. Gentle (not vigorous) bubbling should be noted in the suction control chamber. A total of 50 mL of drainage is not excessive in a client returning to the nursing unit from the recovery room. Drainage that is more than 70 to 100 mL/hour is considered excessive and requires notification of the health care provider. The chest tube insertion site is covered with an occlusive (airtight) dressing to prevent air from entering the pleural space. Positioning the drainage system below the client's chest allows gravity to drain the pleural space.

The nurse is making morning rounds to assess assigned patients. Of the following patients with history of asthma, which patient is of highest priority to assess first? a. A 35-year-old patient who has a longer expiratory phase than inspiratory phase b. A 25-year-old patient with a heart rate of 110 beats/min c. A 42-year-old patient with an oxygen saturation level of 91% at rest d. A 66-year-old patient with a barrel chest and clubbed fingernails

A. A 35-year-old patient who has a longer expiratory phase than inspiratory phase

A nurse assesses patients for potential endocrine disorders. Which patient is at greatest risk for hyperparathyroidism? a. A 72-year-old male who is prescribed home oxygen therapy b. A 66-year-old female with moderate heart failure c. A 29-year-old female with pregnancy-induced hypertension d. A 41-year-old male receiving dialysis for end-stage kidney disease

D. A 41-year-old male receiving dialysis for end-stage kidney disease

The nurse is caring for a dyspneic client with decreased breath sounds. The nurse should carry out which intervention to decrease the client's work of breathing? a. Administer the prescribed bronchodilator. b. Instruct the client to limit fluid intake. c. Place a continuous pulse oximeter on the client. d. Place the client in low Fowler's position.

A. Administer the prescribed bronchodilator. Rationale: Administering the prescribed bronchodilator will help to decrease airway resistance, which decreases the work of breathing and should ease the client's dyspnea. The client should be placed in high Fowler's position to maximize chest expansion. Clients with increased mucus production have increased airway resistance, which increases the work of breathing. Thus, fluids should be increased to help liquefy secretions. Placing a continuous pulse oximeter will assist with monitoring the client's condition but will have no effect on the client's work of breathing.

A home care nurse is visiting a client to provide follow-up evaluation and care of a leg ulcer. On removing the dressing from the leg ulcer, the nurse notes that the ulcer is pale and deep and that the surrounding tissue is cool to the touch. The nurse should document that these findings identify which type of ulcer? a. An arterial ulcer b. A stage 1 pressure ulcer c. A venous stasis ulcer d. A vascular ulcer

A. An arterial ulcer Rationale: Arterial ulcers have a pale deep base and are surrounded by tissue that is cool with trophic changes such as dry skin and loss of hair. Arterial ulcers are caused by tissue ischemia from inadequate arterial supply of oxygen and nutrients. A stage 1 ulcer indicates a reddened area with an intact skin surface. A venous stasis ulcer (vascular) has a dark red base and is surrounded by brown skin with local edema. This type of ulcer is caused by the accumulation of waste products of metabolism that are not cleared, as a result of venous congestion.

The nurse must be alert for signs of respiratory acidosis in the client with emphysema, because this individual has a long-term problem with oxygen maintenance and: a. An inability to fully exhale retained CO2 b. There is a loss of carbon dioxide from the body's buffer pool c. Hyperventilation occurs, even if the cause is not physiologic d. Localized tissue necrosis occurs as a result of poor oxygen supply

A. An inability to fully exhale retained CO2

The nurse is providing education to a group of adolescents diagnosed with asthma. The nurse informs the group that which can be triggers for an asthma attack? Select all that apply. a. An upper respiratory infection (URI) b. Exercise c. Hot air d. Non-steroidal anti-inflammatories e. Cold air

A. An upper respiratory infection (URI) B. Exercise D. Non-steroidal anti-inflammatories E. Cold air Rationale: Triggers for asthma include response to the presence of specific allergens; general irritants such as cold air, dry air, or fine airborne particles; microorganisms; and aspirin and other NSAIDs. Increased airway sensitivity (hyperresponsiveness) can occur with exercise, with an upper respiratory illness, and for unknown reasons. Clean air and adequate rest and sleep help to promote lung function.

A patient is 4 hours postoperative after a femoral-popliteal bypass. The patient reports throbbing leg pain on the affected side, rated as 7/10. What action by the nurse takes priority? a. Assess distal pulses and skin color. b. Notify the surgeon immediately. c. Administer pain medication as ordered. d. Document the findings in the patient's chart.

A. Assess distal pulses and skin color.

A nurse is caring for a patient with a history of renal insufficiency who is scheduled for a cardiac catheterization. What actions would the nurse take prior to the catheterization? (Select all that apply.) a. Assess for allergies to iodine. b. Administer intravenous fluids. c. Administer a prophylactic antibiotic. d. Assess blood urea nitrogen (BUN) and creatinine results. e. Insert a Foley catheter.

A. Assess for allergies to iodine. B. Administer intravenous fluids D. Assess blood urea nitrogen (BUN) and creatinine results.

A client with chronic obstructive pulmonary disease (COPD) is experiencing exacerbation of the disease. The nurse should determine that which finding documented in the client's record is an expected finding with this client? a. Hyperinflation of lungs documented by chest x-ray b. Increased oxygen saturation with ambulation c. A shortened expiratory phase of the respiratory cycle d. A widened diaphragm documented by chest x-ray

A. Hyperinflation of lungs documented by chest x-ray Rationale: The clinical manifestations of COPD are several, including hypoxemia, hypercapnia, dyspnea on exertion and at rest, oxygen desaturation with exercise, use of accessory respiratory muscles, and prolonged exhalation. Chest x-ray results indicate a hyperinflated chest and may indicate a flattened diaphragm if the disease is advanced. Test-Taking Strategy: Focus on the subject, expected finding in COPD. Eliminate option 1 because the reverse is true, even in the client with no respiratory disorder. Eliminate option 2 because it is the opposite of what happens with exacerbation. Eliminate option 4 because the client with COPD has a prolonged expiratory phase.

A nurse teaches a patient with hyperthyroidism. Which dietary modifications should the nurse include in this patient's teaching? (Select all that apply.) a. Increased calorie intake b. Increased carbohydrates c. Decreased fats d. Increased proteins

A. Increased calorie intake B. Increased carbohydrates D. Increased proteins

A client with a history of asthma comes to the emergency department complaining of itchy skin and shortness of breath after starting a new antibiotic. What is the first action the nurse should take? a. Assess for anaphylaxis and prepare for emergency treatment. b. Obtain an arterial blood gas and immunoglobulin E (IgE) blood level. c. Teach the client about the relationship between asthma and allergies. d. Place the client on 100% oxygen and prepare for intubation.

A. Assess for anaphylaxis and prepare for emergency treatment. Rationale: Hypersensitivity or allergy is excessive inflammation occurring in response to the presence of an antigen to which the person usually has been previously exposed. If a client is experiencing an allergic or hypersensitivity response, the nurse's initial action is to assess for anaphylaxis. Promptly notifying the health care provider and preparing emergency equipment, including medication such as epinephrine and possible corticosteroids, is essential in preventing progression of anaphylaxis. Laboratory work is not a priority in this situation. The nurse would expect the IgE level to be elevated; the client may be hypoxic. The nurse would give the client supplemental oxygen; however, 100% is not given unless prescribed, and based on the information in the question intubation is not the first thing the nurse would prepare this client for. Teaching the client is important; however, this is not the right time. When the client is stabilized, the nurse should teach or reinforce that allergies, including some medications, are common triggers for asthma attacks and that people with asthma are predisposed to more allergies than people without asthma.

A nurse assesses a patient with anterior pituitary hyperfunction. Which clinical manifestations would the nurse expect? (Select all that apply.) a. Barrel-shaped chest b. High-pitched voice c. Protrusion of the lower jaw d. Enlarged hands and feet

A. Barrel-shaped chest C. Protrusion of the lower jaw D. Enlarged hands and feet

Your patient, who is post-op from a gastrointestinal surgery, is presenting with a temperature of 103.6 'F, heart rate 120, blood pressure 72/42, increased white blood cell count, and respirations of 21. An IV fluid bolus is ordered STAT. Which findings below indicate that the patient is progressing to septic shock? Select all that apply: a. Blood pressure of 70/34 after the fluid bolus b. Serum lactate less than 2 mmol/L [LOW] c. Patient needs Norepinephrine [vasopressor] to maintain a mean arterial pressure (MAP) greater than 65 mmHg despite fluid replacement d. Central venous pressure (CVP) of 18 [HIGH]

A. Blood pressure of 70/34 after the fluid bolus C. Patient needs Norepinephrine [vasopressor] to maintain a mean arterial pressure (MAP) greater than 65 mmHg despite fluid replacement Rationale: To know if the patient is progressing to septic shock, you need to think about the hallmark findings associated with this condition. Septic shock is characterized by major persistent hypotension (<90 SBP) that doesn't respond to IV fluids (refractory hypotension), and the patient needs vasopressors (ex: Norepinephrine) to maintain a mean arterial pressure greater than 65 and their serum lactate is greater than 2 mmol/L. A serum lactate greater than 2 indicates the cell's tissue/organs are not functioning properly due to low oxygen; hence tissue perfusion is poor due to the low blood pressure and mean arterial pressure.

A client with chronic obstructive pulmonary disease (COPD) is being evaluated for lung transplantation. The nurse performs the initial physical assessment. Which findings should the nurse anticipate in this client? Select all that apply. a. Clubbed fingers b. Muscle retractions c. Increased body temperature d. Prolonged expiratory breathing phase e. Dyspnea at rest f. Decreased respiratory rate

A. Clubbed fingers B. Muscle retractions D. Prolonged expiratory breathing phase E. Dyspnea at rest Rationale: The client with COPD who is eligible for a lung transplantation has end-stage COPD and will have clinical manifestations of hypoxemia, dyspnea at rest, use of accessory muscle with retractions, clubbing, and prolonged expiratory breathing phase caused by retention of carbon dioxide. Option 4 is not correct because the client with COPD has an increased respiratory rate, not a decreased one. Option 5 is not correct because an elevated temperature would not be present unless the client has an infection. Test-Taking Strategy: Focus on the subject, manifestations in COPD for a client with disease advanced enough to be eligible for a lung transplantation. Think about the pathophysiology associated with COPD. Remember that fever is not present with COPD unless there is an infection and that the respiratory rate is normally increased.

A nurse is assessing a patient with left-sided heart failure. For which clinical manifestations would the nurse assess? (Select all that apply.) a. Confusion, restlessness b. Dependent edema c. Cough that worsens at night d. Pulmonary crackles e. Pulmonary hypertension

A. Confusion, restlessness C. Cough that worsens at night D. Pulmonary crackles

Which client is at risk for the development of a sodium level at 130 mEq/L (130 mmol/L)? a. The client who is taking diuretics b. The client with hyperaldosteronism c. The client with Cushing's syndrome

ANS: A Rationale: The normal serum sodium level is 135 to 145 mEq/L (135 to 145 mmol/L). A serum sodium level of 130 mEq/L (130 mmol/L) indicates hyponatremia. Hyponatremia can occur in the client taking diuretics. The client taking corticosteroids and the client with hyperaldosteronism or Cushing's syndrome are at risk for hypernatremia. Test-Taking Strategy: Focus on the subject, the causes of a sodium level of 130 mEq/L (130 mmol/L). First, determine that the client is experiencing hyponatremia. Next, you must know the causes of hyponatremia to direct you to the correct option. Also, recall that when a client takes a diuretic, the client loses fluid and electrolytes.

A client with a diagnosis of diabetic ketoacidosis (DKA) is being treated in the emergency department. Which findings support this diagnosis? Select all that apply. a. Deep, rapid breathing b. Increased pH c. Elevated blood glucose d. Decreased urine output

A. Deep, rapid breathing C. Elevated blood glucose Rationale: Because of the profound deficiency of insulin associated with DKA, glucose cannot be used for energy and the body breaks down fat as a secondary source of energy. Ketones, which are acid byproducts of fat metabolism, build up and the client experiences a metabolic ketoacidosis. High serum glucose contributes to an osmotic diuresis and the client becomes severely dehydrated. If untreated, the client will become comatose due to severe dehydration, acidosis, and electrolyte imbalance. Kussmaul's respirations, the deep rapid breathing associated with DKA, is a compensatory mechanism by the body. The body attempts to correct the acidotic state by blowing off carbon dioxide (CO2), which is an acid. In the absence of insulin, the client will experience severe hyperglycemia. Option 1 is incorrect because in acidosis the pH would be low. Option 4 is incorrect because a high serum glucose will result in an osmotic diuresis and the client will experience polyuria. Test-Taking Strategy: Focus on the subject, findings associated with DKA. Recall that the pathophysiology of DKA is the breakdown of fats for energy. The breakdown of fats leads to a state of acidosis. The high serum glucose contributes to an osmotic diuresis. Knowing the pathophysiology of DKA will aid in identification of the correct answer.AC

The nurse is monitoring the chest tube drainage system in a client with a chest tube. The nurse notes intermittent bubbling in the water seal chamber. Which is the most appropriate nursing action? a. Document the findings. b. Check for an air leak. c. Notify the health care provider. d. Change the chest tube drainage system

A. Document the findings Rationale: Bubbling in the water seal chamber is caused by air passing out of the pleural space into the fluid in the chamber. Intermittent (not constant) bubbling is normal. It indicates that the system is accomplishing one of its purposes, removing air from the pleural space. Continuous bubbling during inspiration and expiration indicates that an air leak exists. If this occurs, it must be corrected. Notifying the health care provider and changing the chest tube drainage system are not indicated at this time. Test-Taking Strategy: Note the strategic words, most appropriate. Note the subject, chest tube drainage systems, and focus on the words intermittent bubbling and water seal chamber. Recalling that intermittent (not constant) bubbling is normal in this chamber will direct you to the correct option.

Which nursing interventions are appropriate in caring for a client with emphysema? Select all that apply. a. Encourage alternating activity with rest periods. b. Reduce fluid intake to less than 1500 mL/day. c. Teach the client techniques of chest physiotherapy. d. Teach diaphragmatic and pursed-lip breathing. e. Keep the client in a supine position as much as possible.

A. Encourage alternating activity with rest periods. C. Teach the client techniques of chest physiotherapy. D. Teach diaphragmatic and pursed-lip breathing. Rationale: Fluids are encouraged, not reduced, to liquefy secretions for easier expectoration. Diaphragmatic and pursed-lip breathing assists in opening alveoli and eases dyspnea. The client should be encouraged to perform activities and exercise, such as dressing and walking, as tolerated with rest periods in between. Chest physiotherapy consists of percussion, vibration, and postural drainage. These techniques are helpful in removing secretions. Elevating the head of the bed assists with breathing.

The nurse is caring for a client who is on strict bed rest and creates a plan of care with goals related to the prevention of deep vein thrombosis and pulmonary emboli. Which nursing action is most helpful in preventing these disorders from developing? a. Encouraging active range-of-motion exercises b. Restricting fluids c. Applying a heating pad to the lower extremities d. Placing a pillow under the knees

A. Encouraging active range-of-motion exercises Rationale: Clients at greatest risk for deep vein thrombosis and pulmonary emboli are immobilized clients. Basic preventive measures include early ambulation, leg elevation, active leg exercises, elastic stockings, and intermittent pneumatic calf compression. Keeping the client well hydrated is essential because dehydration predisposes to clotting. A pillow under the knees may cause venous stasis. Heat should not be applied without a health care provider's prescription.

True or False: Septic shock causes system wide vasodilation which leads to an increase in systemic vascular resistance. In addition, septic shock causes increased capillary permeability and clot formation in the microcirculation throughout the body. a. FALSE b. TRUE

A. False Rationale: The correct answer is FALSE. This statement is incorrect because there is a DECREASE (not increased) systemic vascular resistance in septic shock due to vasodilation. In septic shock, vasodilation is system wide. In addition, septic shock causes increased capillary permeability and thrombi formation in the microcirculation throughout the body. The vasodilation, increased capillary permeability, and clot formation in the microcirculation all leads to a decrease in tissue perfusion. This causes organ and tissue dysfunction, hence septic shock.

A patient with a severe infection has developed septic shock. The patient's blood pressure is 72/44, heart rate 130, respiration 22, oxygen saturation 96% on high-flow oxygen, and temperature 103.6 'F. The patient's mean arterial pressure (MAP) is 53 mmHg. Based on these findings, you know this patient is experiencing diminished tissue perfusion and needs treatment to improve tissue perfusion to prevent organ dysfunction. In regards to the pathophysiology of septic shock, what is occurring in the body that is leading to this decrease in tissue perfusion? Select all that apply: a. Increased systemic vascular resistance b. A significantly decreased cardiac output c. Vasodilation d. Increased capillary permeability e. Clot formation in microcirculation

A. Increased capillary permeability C. Vasodilation E. Clot formation in microcirculation Rationale: Septic shock occurs due to sepsis. Sepsis is the body's reaction to an infection and will lead to septic shock if this reaction is not treated. This reaction is the activation of the body's inflammatory system, but it's MAJORLY amplified and system wide. Cardiac output is not the problem in septic shock as with other types of shocks like hypovolemic or cardiogenic. CO is actually high or normal during the early stages of septic shock. It only decreases to the end of septic shock when heart function fails. The issue is with what is going on beyond the heart in the vessels. Substances are released by the microorganism that has invaded the body. This causes the immune system to release substances that will cause system wide vasodilation of the vessels (this will cause a DECREASE in systemic vascular resistance, blood to pool, and this decreases blood flow to the organs/tissues) along with an increase in capillary permeability (this causes fluid to leave the intravascular system and depletes the circulatory system of fluid and further decreases blood flow to the organs/fluids...this is RELATIVE (not absolute) hypovolemia). Furthermore, clots will form in the microcirculation due to plasma activating factor being released. This will cause platelets to aggregate and block blood flow even more to the organs/tissues. All of this will lead to decreased tissue perfusion and deprive cells of oxygen.

The nurse is providing preoperative teaching with the client about the use of an incentive spirometer in the postoperative period. Which instructions should the nurse NOT include? a. Inhale as deeply and quickly as possible. b. Place the mouthpiece in your mouth and seal your lips tightly around it. c. After maximum inspiration, hold the breath for 2 to 3 seconds and exhale. d. Sit upright in the bed or in a chair.

A. Inhale as deeply and quickly as possible

The nurse is reinforcing instructions to a client about the use of an incentive spirometer. The nurse tells the client to sustain the inhaled breath for 3 seconds. When the client asks the nurse about the rationale for this action, the nurse explains that which is the primary benefit? a. Maintain inflation of the alveoli. b. Enhance ciliary action in the tracheobronchial tree. c. Increase surfactant production d. Dilate the major bronchi.

A. Maintain inflation of the alveoli Rationale: Sustained inhalation when using an incentive spirometer helps maintain inflation of the terminal bronchioles and alveoli, thereby promoting better gas exchange. Routine use of devices such as an incentive spirometer can help prevent atelectasis and pneumonia in clients at risk. The remaining options are not benefits for sustained inhalation. Test-Taking Strategy: Focus on the subject, the benefit of sustaining the inhaled breath. Also, note the strategic word, primary, indicating that all of the options may be correct but one option is the best benefit. Use knowledge of anatomy and physiology. Recalling that the alveoli are the most distal part of the respiratory tree will direct you to the correct option.

A client with a chest injury has suffered flail chest. The nurse assesses the client for which most distinctive sign of flail chest? a. Paradoxical chest movement b. Hypotension c. Dyspnea, especially on exhalation d. Cyanosis

A. Paradoxical chest movement Rationale: Flail chest results from multiple rib fractures. This results in a "floating" section of ribs. Because this section is unattached to the rest of the bony rib cage, this segment results in paradoxical chest movement. This means that the force of inspiration pulls the fractured segment inward, while the rest of the chest expands. Similarly, during exhalation, the segment balloons outward while the rest of the chest moves inward. This is a characteristic sign of flail chest. Test-Taking Strategy: Note the strategic word, most. Cyanosis and hypotension occur with many different disorders, so eliminate options 1 and 2 first. From the remaining options, choose paradoxical chest movement over dyspnea on exhalation by remembering that a flail chest has broken rib segments that move independently of the rest of the rib cage

The nurse is assessing four hospitalized clients for fluid volume deficit. Which client should the nurse assess further as the priority? a. 102 kg client; urine output 73 ml in 1 hour b. 98 kg client; urine specific gravity 1.042 (high) c. 106 kg client; pulse 108 beats per minute d. 79 kg client; cannot obtain fluids

ANS: B Rationale: Urine specific gravity should be 1.010-1.030. A client whose urine specific gravity is 1.042 (high = concentrated urine) has probable fluid volume deficit. An output of 73 ml an hour is normal for a 102 kg client. A pulse of 108 is slightly high but may not be specifically due to fluid volume deficit. The client who cannot obtain fluids should be put on "fluid rounds" but there is no data to indicate this client has fluid volume deficit already.

Your patient is started on an IV antibiotic to treat a severe infection. During infusion, the patient uses the call light to notify you that she feels a tight sensation in her throat and it's making it hard to breathe. You immediately arrive to the room and assess the patient. While auscultating the lungs you note wheezing. You also notice that the patient is starting to scratch the face and arms, and on closer inspection of the face you note redness and swelling that extends down to the neck and torso. The patient's vital signs are the following: blood pressure 89/62, heart rate 118 bpm, and oxygen saturation 88% on room air. You suspect anaphylactic shock. Select all the appropriate interventions for this patient: a. Prepare for the administration of Epinephrine b. Place the patient on oxygen c. Slow down the antibiotic infusion d. Call a rapid response

A. Prepare for the administration of Epinephrine B. Place the patient on oxygen D. Call a rapid response Rationale: Option C is wrong because the nurse should STOP the infusion, not slow it down because this could be the reason for the anaphylactic reaction. The nurse would want to call a rapid response, place the patient on oxygen, and prepare for the administration of Epinephrine. This drug is the first-line treatment for anaphylactic shock. It will increase the blood pressure, decrease swelling, and dilate the airway.

The nurse is planning care for an 81-year-old unresponsive client admitted to the hospital with a medical diagnosis of pneumonia. The nurse has identified the problem of inability to clear the airway related to retained secretions. Which intervention is most appropriate? a. Provide nasotracheal suctioning as needed to remove secretions. b. Plan activities with rest periods to conserve oxygen needs. c. Monitor oxygenation (the oxygen saturation [SaO2]) during activity. d. Initiate and maintain supplemental oxygen as prescribed.

A. Provide nasotracheal suctioning as needed to remove secretions Rationale: Note that the patient is "unresponsive", therefor unable to cough and deep breath to clear secretions on own. Ineffective airway clearance reflects the client's inability to expectorate secretions. The intervention specifically addressing retained secretions is in the correct option. Options 1 and 4 are interventions addressing impaired problem with gas exchange. Option 2 is an intervention aimed at addressing a problem with activity intolerance. Test-Taking Strategy: Note the subject, care of the client with pneumonia. Also, note the strategic words, most appropriate. Focus on the data in the question. Note the relationship between the words inability to clear the airway related to retained secretions in the question and the correct option.

A nurse cares for a patient who is recovering from a pituitary gland resection (hypophysectomy). What action would the nurse take first? a. Report clear or light yellow drainage from the nose. b. Instruct the patient to cough, turn, and deep breathe. c. Apply petroleum jelly to lips to avoid dryness d. Keep the head of the bed flat and the patient supine.

A. Report clear or light yellow drainage from the nose

A client with no history of heart disease has experienced acute myocardial infarction and has been given thrombolytic therapy with tissue plasminogen activator (TPA). What assessment finding should the nurse identify as an indicator that the client is experiencing complications of this therapy? a. Tar-like stools b. Nausea and vomiting c. Decreased urine output d. Orange-colored urine

A. Tar-like stools Rationale: Thrombolytic agents are used to dissolve existing thrombi, and the nurse should monitor the client for obvious or occult signs of bleeding. This includes assessment for obvious bleeding within the gastrointestinal (GI) tract, urinary system, and skin. It also includes Hematest testing of secretions for occult blood. The correct option is the only one that indicates the presence of blood. Test-Taking Strategy: Focus on the subject, complications of thrombolytic therapy. Note the word thrombolytic, meaning to dissolve clots, and focus your attention on blood coagulation. Look for the option that has a hematological connection, in this case, tar-like stools, which indicate bleeding from the GI tract.

The nurse instructs a client on pursed-lip breathing and asks the client to demonstrate the breathing technique. Which observation by the nurse would indicate that the client is performing the technique correctly? a. The client breathes out slowly through the mouth. b. The client breathes in through the mouth. c. The client avoids using the abdominal muscles to breathe out. d. The client puffs out the cheeks when breathing out through the mouth

A. The client breathes out slowly through the mouth Rationale: Pursed-lip breathing facilitates maximal expiration for clients with obstructive lung disease. The client should close the mouth and breathe in through the nose. The client then purses the lips and breathes out slowly through the mouth, without puffing the cheeks. The client should spend at least twice the amount of time breathing out that it took to breathe in. The client should use the abdominal muscles to assist in squeezing out all of the air. The client also is instructed to use this technique during any physical activity, inhale before beginning the activity, and exhale while performing the activity. The client is also instructed that he or she should never hold the breath.

The nurse is caring for a hospitalized client who is retaining carbon dioxide (CO2) because of respiratory disease. The nurse anticipates which physical response will initially occur? a. The client's arterial blood gas results will reflect acidosis. b. The client's sodium and chloride levels will rise. c. The client will complain of facial numbness and tingling. d. The client will lose consciousness.

A. The client's arterial blood gas results will reflect acidosis Rationale: When the client with respiratory disease retains CO2, a rise in CO2 will occur. This results in a corresponding fall in pH, thus respiratory acidosis. This concept forms the basis for key aspects of acid-base balance. The other options are incorrect and are not associated with this initial physical response. Test-Taking Strategy: Note the strategic word, initially. Note the words retaining carbon dioxide, and focus on the subject, the client's diagnosis of respiratory disease. Recalling the inverse relationship between pH and CO2 in the body will direct you to the correct option. As CO2 rises, pH falls, and as CO2 falls, pH rises.

You're assessing the patient's complete blood count (CBC). Which lab result below demonstrates leukopenia? a. WBC 3,000 b. WBC 7,000 c. Platelets 90,000 d. Platelets 500,000

A. WBC 3,000 Rationale: WBC range is 5,000-10,000. Leukopenia is a DECREASED white blood count.

The nurse is planning to teach a client with peripheral arterial disease about measures to limit disease progression. Which items should the nurse include on a list of suggestions for the client? Select all that apply. a. Walk each day to increase circulation to the legs. b. Soak the feet in hot water daily. c. Be careful not to injure the legs or feet. d. Cut down on the amount of fats consumed in the diet. e. Use a heating pad on the legs to aid vasodilation.

A. Walk each day to increase circulation to the legs C. Be careful not to injure the legs or feet D. Cut down on the amount of fats consumed in the diet Rationale: Long-term management of peripheral arterial disease consists of measures that increase peripheral circulation (exercise), promote vasodilation (warmth), relieve pain, and maintain tissue integrity (foot care and nutrition). Soaking the feet in hot water and application of a heating pad to the extremity are contraindicated. The affected extremity may have decreased sensitivity and is at risk for burns. Also, the affected tissue does not obtain adequate circulation at rest. Direct application of heat raises oxygen and nutritional requirements of the tissue even further. Test-Taking Strategy: Note the subject, teaching for the client with peripheral arterial disease. Focus on the client's diagnosis and the words measures to limit disease progression. Recalling that the client is at risk for altered tissue integrity will assist you in answering correctly. Also, note the words hot and heating in the incorrect options, recalling that this is a risk for injury.

The nurse is reviewing the health care provider's (HCP's) prescriptions for a client with a diagnosis of diabetes mellitus who has been hospitalized for treatment of an infected foot ulcer. The nurse expects to note which finding in the HCP's prescriptions? a. An increased amount of NPH insulin daily insulin b. A decreased-calorie diet c. An increased-calorie diet d. A decreased amount of NPH insulin daily insulin

ANS: A Rationale: Infection is a physiological stressor that can cause an increase in the level of cortisol in the body. An increase in cortisol causes an increase in blood glucose levels. When the client is under stress, such as when an infection is present, an increase in the dose of insulin will be required to facilitate the transport of excess glucose into the cells. The client will not necessarily need an adjustment in the daily diet Test-Taking Strategy: Focus on the subject, care of the client with diabetes mellitus. Noting that the client has an infected foot ulcer will indicate that a stressor is present. Recalling that a stressor increases the client's need for insulin will direct you to the correct option.

A client has fluid volume deficit and the provider has prescribed isotonic IV solution at a rate of 100 ml/hour. Which solution does the nurse choose? a. 0.9% sodium chloride (NS) b. 5% dextrose in water (D5W) c. 0.45% sodium chloride (1/2 NS) d. 10% dextrose in water (D10W)

ANS: A Rationale: A 0.9% sodium chloride is the only isotonic solution listed. 0.45% NS and D5W are hypotonic. D5W is often listed as isotonic, but when infusing, the dextrose is metabolized so quickly that in essence it is hypotonic. D10W is then hypertonic.

The nurse is assessing a client who had abdominal surgery earlier in the day. Which preexisting medical condition would place the client at most risk for postoperative complications? a. Alcohol Abuse b. Osteoporosis c. Pacemaker d. Peptic Ulcer Disease

ANS: A Rationale: A client with a history of alcohol abuse is at risk for liver disease, including altered metabolism and elimination of medications, impaired wound healing, and clotting and bleeding abnormalities. A client with this risk factor also would be at risk for experiencing alcohol withdrawal during the postoperative period. Clients with a pacemaker, osteoporosis, and peptic ulcer disease need to be monitored closely but are not at risk for major complications, as is the client with alcohol abuse and liver disease. Test-Taking Strategy: Note the strategic word, most, and focus on the subject, the client at risk for complications after abdominal surgery. Thinking about the pathophysiological alterations that occur as a result of alcohol abuse will direct you to the correct option.

The nurse is caring for a client who recently returned from the operating room. On data collection, the nurse notes that the client's vital signs are blood pressure (BP), 118/70 mm Hg; pulse, 91 beats/minute; and respirations, 16 breaths/minute. Preoperative vital signs were BP, 132/88 mm Hg; pulse, 74 beats/minute; and respirations, 20 breaths/minute. Which action should the nurse plan to take first? a. Recheck the vital signs in 15 minutes. b. Call the surgeon immediately. c. Shake the client gently to arouse. d. Cover the client with a warm blanket

ANS: A Rationale: A drop in blood pressure slightly below a client's preoperative baseline reading is common after surgery, especially considering medications given intra-op. The nurse should recheck the vital signs frequently post-operatively. Warm blankets are applied to maintain the client's body temperature, but this is not what needs to be done first. Level of consciousness can be assessed by the evaluation of the client's response to light touch and verbal stimuli. It is not necessary to contact the surgeon immediately. Test-Taking Strategy: Note the strategic word, first. Focus on the data in the question, and note that the vital signs are within normal limits following a surgical procedure; therefore, the nurse should plan to recheck the vital signs.

The nurse is assigned to care for a group of clients. On review of the clients' medical records, the nurse determines that which client is most likely at risk for a fluid volume deficit? a. A client with an ileostomy b. A client with heart failure c. A client on long-term corticosteroid therapy d. A client receiving frequent wound irrigations

ANS: A Rationale: A fluid volume deficit occurs when the fluid intake is not sufficient to meet the fluid needs of the body. Causes of a fluid volume deficit include vomiting, diarrhea, conditions that cause increased respirations or increased urinary output, insufficient intravenous fluid replacement, draining fistulas, and the presence of an ileostomy or colostomy. A client with heart failure or on long-term corticosteroid therapy or a client receiving frequent wound irrigations is most at risk for fluid volume excess. Test-Taking Strategy: Note the strategic words, most likely. Read the question carefully, noting the subject, the client at risk for a deficit. Read each option and think about the fluid imbalance that can occur in each. The clients with heart failure, on long-term corticosteroid therapy, and receiving frequent wound irrigations retain fluid. The only condition that can cause a deficit is the condition noted in the correct option.

The nurse is caring for a client with a diagnosis of severe dehydration. The client has been receiving intravenous (IV) fluids and nasogastric (NG) tube feedings. The nurse monitors fluid balance using which as the best indicator? a. Daily weight b. Urinary output c. IV fluid intake d. NG tube intake

ANS: A Rationale: Daily weight is the best indicator of fluid balance. Options 2, 3, and 4 are related to intake or output but are incomplete indicators of fluid balance. Test-Taking Strategy: Note the strategic word, best. Remember that the best indicator of fluid balance is weight. Also note that options 2, 3, and 4 are comparable or alike. These options represent measurements of intake or output.

The nurse provides instructions to a client newly diagnosed with type 1 diabetes mellitus. The nurse recognizes accurate understanding of measures to prevent diabetic ketoacidosis when the client makes which statement? a. "I will notify my health care provider (HCP) if my blood glucose level is higher than 250 mg/dL (14.2 mmol/L)." b. "I will stop taking my insulin if I'm too sick to eat." c. "I will decrease my insulin dose during times of illness." d. "I will adjust my insulin dose according to the level of glucose in my urine."

ANS: A Rationale: During illness, the client with type 1 diabetes mellitus is at increased risk of diabetic ketoacidosis, due to hyperglycemia associated with the stress response and due to a typically decreased caloric intake. As part of sick day management, the client with diabetes should monitor blood glucose levels and should notify the HCP if the level is higher than 250 mg/dL (14.2 mmol/L). Insulin should never be stopped. In fact, insulin may need to be increased during times of illness. Doses should not be adjusted without the HCP's advice and are usually adjusted on the basis of blood glucose levels, not urinary glucose readings. Test-Taking Strategy: Use general medication guidelines to answer the question. Note that options 1, 2, and 3 are comparable or alike and all relate to adjustment of insulin doses.

A client with diabetes mellitus is being discharged following treatment for hyperosmolar hyperglycemic syndrome (HHS) precipitated by acute illness. The client tells the nurse, "I will call the health care provider (HCP) the next time I can't eat for more than a day or so." Which statement reflects the most appropriate analysis of this client's level of knowledge? a. The client needs immediate education before discharge. b. The client's statement is inaccurate, and he or she should be scheduled for outpatient diabetic counseling. c. The client requires follow-up teaching regarding the administration of oral antidiabetics. d. The client's statement is inaccurate, and he or she should be scheduled for educational home health visits.

ANS: A Rationale: If the client becomes ill and cannot retain fluids or food for a period of 4 hours, the HCP should be notified. The client's statement indicates a need for immediate education to prevent hyperosmolar hyperglycemic syndrome (HHS), a life-threatening emergency. Although all of the other options may be true, the most appropriate analysis is that the client requires immediate education. Test-Taking Strategy: Note the strategic words, most appropriate. Focus on the subject, client instructions to prevent HHS. Eliminate options that are not immediately threatening. The client requires immediate education.

The nurse is caring for a client admitted to the emergency department with diabetic ketoacidosis (DKA). In the acute phase, the nurse plans for which priority intervention? a. Administer short-duration insulin intravenously. b. Administer 5% dextrose intravenously. c. Apply a monitor for an electrocardiogram. d. Correct the acidosis

ANS: A Rationale: Lack of insulin (absolute or relative) is the primary cause of DKA. Treatment consists of insulin administration (short- or rapid-acting), intravenous fluid administration (normal saline initially, not 5% dextrose), and potassium replacement, followed by correcting acidosis. Cardiac monitoring is important due to alterations in potassium levels associated with DKA and its treatment, but applying an electrocardiogram monitor is not the priority action. Test-Taking Strategy: Focus on the client's diagnosis. Note the strategic word, priority. Remember that in DKA, the initial treatment is short- or rapid-acting insulin. Normal saline is administered initially; therefore, option 2 is incorrect. Options 1 and 3 may be components of the treatment plan but are not the priority.

The nurse is teaching a client about coughing and deep-breathing techniques to prevent postoperative complications. Which statement is most appropriate for the nurse to make to the client at this time as it relates to these techniques? a. "Use of an incentive spirometer will help prevent pneumonia." b. "Close monitoring of your oxygen saturation will detect hypoxemia." c. "Administration of intravenous fluids will prevent or treat fluid imbalance." d. "Early ambulation and administration of blood thinners will prevent pulmonary embolism."

ANS: A Rationale: Postoperative respiratory problems are atelectasis, pneumonia, and pulmonary emboli. Pneumonia is the inflammation of lung tissue that causes productive cough, dyspnea, and lung crackles and can be caused by retained pulmonary secretions. Use of an incentive spirometer helps to prevent pneumonia and atelectasis. Hypoxemia is an inadequate concentration of oxygen in arterial blood. While close monitoring of the oxygen saturation will help to detect hypoxemia, monitoring is not directly related to coughing and deep-breathing techniques. Fluid imbalance can be a deficit or excess related to fluid loss or overload, and surgical clients are often given intravenous fluids to prevent a deficit; however, this is not related to coughing and deep breathing. Pulmonary embolus occurs as a result of a blockage of the pulmonary artery that disrupts blood flow to 1 or more lobes of the lung; this is usually due to clot formation. Early ambulation and administration of blood thinners helps to prevent this complication; however, it is not related to coughing and deep-breathing techniques.

During an assessment of a newly admitted client, the nurse notes that the client's heart rate is 110 beats/minute, his blood pressure shows orthostatic changes when he stands up, and his tongue has a sticky, paste-like coating. The client's spouse tells the nurse that he seems a little confused and unsteady on his feet. Based on these assessment findings, the nurse suspects that the client most likely has which condition? a. Dehydration b. Hypokalemia c. Fluid Overload d. Hypernatremia

ANS: A Rationale: When a client is dehydrated, the heart rate increases in an attempt to maintain blood pressure. Blood pressure reflects orthostatic changes caused by the reduced blood volume, and when the client stands, he may experience dizziness because of insufficient blood flow to the brain. Alterations in mental status also may occur. The oral mucous membranes, usually moist, are dry and may be covered with a thick, pasty coating. Hypernatremia may be a result of dehydration, but a patient could be hypernatremic for reasons other than dehydration. Test-Taking Strategy: Focus on the subject, the condition characteristic of the findings noted in the question. Focus on the data in the question—heart rate of 110 beats/minute, orthostatic blood pressure changes, and sticky, paste-like coating on the tongue. Think about the manifestations that would occur in each condition noted in the options. These assessment findings reflect dehydration, or fluid volume deficit.

A client has been diagnosed with hyperthyroidism. The nurse monitors for which signs and symptoms indicating a complication of this disorder? Select all that apply. a. Nausea b. Fever c. Confusion d. Bradycardia e. Lethargy f. Tremors

ANS: A, B, C, F Rationale: Thyroid storm is an acute and life-threatening complication that occurs in a client with uncontrollable hyperthyroidism. Signs and symptoms of thyroid storm include elevated temperature (fever), nausea, and tremors. In addition, as the condition progresses, the client becomes confused. The client is restless and anxious and experiences tachycardia. Test-Taking Strategy: Focus on the subject, signs and symptoms indicating a complication of hyperthyroidism. Recall that thyroid storm is a complication of hyperthyroidism. Options 3 and 6 can be eliminated if you remember that thyroid storm is caused by the release of thyroid hormones into the bloodstream, causing uncontrollable hyperthyroidism. Lethargy and bradycardia (think: slow down) are signs of hypothyroidism (slow metabolism).

The nurse is preparing a client with a new diagnosis of hypothyroidism for discharge. The nurse determines that the client understands discharge instructions if the client states that which signs and symptoms are associated with this diagnosis? Select all that apply. a. Feeling cold b. Loss of body hair c. Weight loss d. Persistent lethargy e. Tremors

ANS: A, B, D Rationale: Feeling cold, hair loss, lethargy, and facial puffiness are signs of hypothyroidism. Tremors and weight loss are signs of hyperthyroidism. Test-Taking Strategy: Focus on the subject, signs and symptoms associated with hypothyroidism. Options 1 and 2 can be eliminated if you remember that in hypothyroidism there is an undersecretion of thyroid hormone that causes the metabolism to slow down.

The nurse provides instructions to a client with a low potassium level about the foods that are high in potassium and tells the client to consume which foods? Select all that apply. a. Bananas b. Potatoes c. Peas d. Cantaloupe e. Cauliflower

ANS: A, B, D Rationale: The normal potassium level is 3.5 to 5.0 mEq/L (3.5 to 5.0 mmol/L). Common food sources of potassium include avocado, bananas, cantaloupe, carrots, fish, mushrooms, oranges, potatoes, pork, beef, veal, raisins, spinach, strawberries, and tomatoes. Peas and cauliflower are high in magnesium. Test-Taking Strategy: Focus on the subject, foods high in potassium. Read each food item and use knowledge about nutrition and components of food. Recall that peas and cauliflower are high in magnesium.

The nurse is caring for a client with heart failure (HF). Which signs and symptoms could indicate fluid overload? Select all that apply. a. Bounding pulse b. Difficulty breathing c. Increased during output d. Presence of dependent edema e. Neck vein distention in the upright position

ANS: A, B, D, E Rationale: Care of a client with HF and fluid overload includes monitoring for bounding pulses, difficulty breathing, neck vein distention in the upright position, and dependent edema. Increased urine output is not associated with HF and fluid overload. Test-Taking Strategy: Note the subject of the question, signs and symptoms of fluid overload. Eliminate option 3 because decreased, not increased, urine output is related to fluid overload.

The nurse is caring for a client who needs a hypertonic intravenous (IV) solution. What solutions are hypertonic? Select all that apply. a. 10% dextrose in water b. 0.45% sodium chloride c. 5% dextrose in 0.9% saline d. 5% dextrose in 0.45% saline e. 0.9% Normal Saline

ANS: A, C, D Rationale: Hypertonic fluids include 10% dextrose in water, 5% dextrose in 0.9% saline, 5% dextrose in 0.45% saline, and 5% dextrose in lactated Ringer's solution. The solutions of 0.45% sodium chloride and 5% dextrose in 0.225% saline are not hypertonic solutions. Test-Taking Strategy: Focus on the subject, hypertonic solutions. Hypertonic solutions have a higher osmolality than do body fluids and are used to correct fluid, electrolyte, and acid-base imbalances by moving water out of the body's cells and into the bloodstream. Eliminate option 2 because this is a hypotonic solution

A client is admitted to a surgical unit postoperatively with a wound drain in place. Which actions should the nurse take in the care of the drain? Select all that apply. a. Maintain aseptic technique when emptying the drain. b. Clamp the drain for 15 minutes every hour. c. Curl the drain tightly, and tape it firmly to the body. d. Check the drain for patency. e. Observe for bright red bloody drainage.

ANS: A, D, E Rationale: The nurse should check the tube or drain for patency to provide an exit for the fluid or blood to promote healing. The nurse should monitor the drainage characteristics. Usually the drainage from the wound is pale, red, and watery. Active bleeding will be bright red. The nurse must use aseptic technique for emptying the drainage container or changing the dressing to avoid contamination of the wound. A postoperative drain should not be curled tightly or obstructed in any way, such as with clamping. This could prevent the drain from functioning properly.

Which assessment data finding for a client scheduled for total knee replacement surgery is most important for the nurse to communicate to the surgeon and the anesthesia provider before the procedure? Select all that apply. a. The serum potassium level is 3.0 mEq/L (3.0 mmol/L). b. The oxygen saturation is 97%. c. The client requests to talk with a registered dietitian about weight loss. d. After receiving the preoperative medications, the client tells the nurse that he lied on the assessment form and that he really is a current smoker. e. The client took a total of 1300 mg of aspirin yesterday.

ANS: A, D, E Rationale: This is a low potassium value (3.0 mEq/L) which should be communicated to the surgeon and anesthesia provider prior to surgery. Taking aspirin prior to surgery can increase the risk for bleeding. This should be communicated to the surgeon and anesthesia provider prior to surgery. The client's smoking status can change important assessment information collected by the surgeon and anesthesia provider; therefore, this should be immediately communicated. The oxygen saturation level is normal, and it is acceptable that the regularly scheduled antihypertensive was taken with a sip of water 2 hours ago.

A client is scheduled for surgery at noon. The surgeon is delayed and the surgery is now scheduled for 3:00 PM. How will the nurse plan to administer the preoperative prophylactic antibiotic? a. Give at noon as originally prescribed. b. Adjust the administration time to be given within one hour prior to surgery. c. Hold the preoperative antibiotic so it can be administered immediately following surgery. d. Cancel orders; preoperative prophylactic antibiotics are given optionally.

ANS: B Rationale: According to the Surgical Care Improvement Project (SCIP) guidelines, prophylactic antibiotics should be given within one hour before the surgical incision.

The nurse is preparing a plan of care for a client with diabetes mellitus who has hyperglycemia. The nurse places priority on which client problem? a. Inadequate consumption of nutrients b. Inadequate fluid volume c. Compromised family coping d. Lack of Knowledge

ANS: B Rationale: An increased blood glucose level will cause the kidneys to excrete the glucose in the urine. This glucose is accompanied by fluids and electrolytes, causing an osmotic diuresis leading to dehydration. This fluid loss must be replaced when it becomes severe. Options 1, 3, and 4 are not related specifically to the information in the question. Test-Taking Strategy: Note the strategic word, priority, and focus on the data in the question. Use Maslow's Hierarchy of Needs theory. The correct option indicates a physiological need and is the priority. Options 1, 3, and 4 are problems that may need to be addressed after providing for the priority physiological needs.

A student nurse is giving hand-off report to the registered nurse on four clients who have fluid volume deficit. Which client should the registered nurse assess first? a. 66 kg client, urine output averages 36ml/hour for the last 4 hours b. 86-year-old client, IV fluids infusing at 100 ml/hour, rales bilaterally c. 76-year-old client, urine specific gravity 1.028 d. 100 kg client, lying BP 128/72 mmHg, standing BP 118/68 mmHg

ANS: B Rationale: During fluid therapy for fluid volume deficit, the nurse should monitor for complications such as heart failure, especially in older clients. A new onset of bilateral rales could indicate heart failure, possibly worsened by the IV infusion rate of 100mL/hr, and the nurse should check this client first. The other clients all have normal findings.

A client with type 1 diabetes mellitus calls the nurse to report recurrent episodes of hypoglycemia with exercising. Which statement by the client indicates an adequate understanding of the peak action of NPH insulin and exercise? a. "The best time for me to exercise is mid- to late afternoon." b. "The best time for me to exercise is after breakfast." c. "I should not exercise since I am taking insulin." d. "NPH is a basal insulin, so I should exercise in the evening."

ANS: B Rationale: Exercise is an important part of diabetes management. It promotes weight loss, decreases insulin resistance, and helps to control blood glucose levels. A hypoglycemic reaction may occur in response to increased exercise, so clients should exercise either an hour after mealtime or after consuming a 10- to 15-gram carbohydrate snack, and they should check their blood glucose level before exercising. Option 1 is incorrect because clients with diabetes should exercise, though they should check with their health care provider before starting a new exercise program. Option 3 in incorrect; clients should avoid exercise during the peak time of insulin. NPH insulin peaks at 4 to 12 hours; therefore, afternoon exercise takes place during the peak of the medication. Options 4 is incorrect; NPH insulin in an intermediate-acting insulin, not a basal insulin. Test-Taking Strategy: Focus on the subject, peak action of NPH insulin. Recalling that NPH insulin peaks at 4 to 12 hours and that exercise is beneficial for clients with diabetes will direct you to the correct option.

The nurse caring for a client with a diagnosis of hypoparathyroidism reviews the laboratory results of blood tests for this client and notes that the calcium level is extremely low. The nurse should expect to note which finding on assessment of the client? a. Unresponsive pupils b. Positive Trousseau's sign c. Negative Chvostek's sign d. Hypoactive bowel sounds

ANS: B Rationale: Hypoparathyroidism is related to a lack of parathyroid hormone secretion or a decreased effectiveness of parathyroid hormone on target tissues. The end result of this disorder is hypocalcemia. When serum calcium levels are critically low, the client may exhibit Chvostek's and Trousseau's signs, which indicate potential tetany. The remaining options are not related to the presence of hypocalcemia. Test-Taking Strategy: Focus on the subject, assessment findings associated with hypoparathyroidism and hypocalcemia. Recalling that positive Chvostek's and Trousseau's signs would be noted in this disorder will direct you to the correct option. Also, noting the word positive in the correct option will assist you in answering correctly.

A client arrives in the hospital emergency department in an unconscious state. As reported by the spouse, the client has diabetes mellitus and began to show symptoms of hypoglycemia. A blood glucose level is obtained for the client, and the result is 40 mg/dL (2.28 mmol/L). Which medication should the nurse anticipate will be prescribed for the client? a. Glyburide b. Metformin c. Glucagon d. Regular insulin

ANS: C Rationale: A blood glucose level lower than 50 mg/dL (2.85 mmol/L) is considered to be critically low. Glucagon is used to treat hypoglycemia because it increases blood glucose levels. Insulin would lower the client's blood glucose and would not be an appropriate treatment for hypoglycemia. Glyburide and metformin are oral hypoglycemic agents used to treat type 2 diabetes mellitus and would not be given to a client with hypoglycemia. In addition, an oral medication would not be administered to an unconscious client. Test-Taking Strategy: Focus on the subject, hypoglycemia. Knowing that insulin, glyburide, and metformin are used to treat hyperglycemia will assist in directing you to the correct option.

The nurse is caring for a client with a nasogastric tube that is attached to low suction. The nurse monitors the client for manifestations of which disorder that the client is at risk for? a. Metabolic acidosis b. Metabolic alkalosis c. Respiratory acidosis d. Respiratory alkalosis

ANS: B Rationale: Metabolic alkalosis is defined as a deficit or loss of hydrogen ions or acids or an excess of base (bicarbonate) that results from the accumulation of base or from a loss of acid without a comparable loss of base in the body fluids. This occurs in conditions resulting in hypovolemia, the loss of gastric fluid, excessive bicarbonate intake, the massive transfusion of whole blood, and hyperaldosteronism. Loss of gastric fluid via nasogastric suction or vomiting causes metabolic alkalosis as a result of the loss of hydrochloric acid. The remaining options are incorrect interpretations. Test-Taking Strategy: Focus on the subject, a client with a nasogastric tube attached to suction. Remembering that a client receiving nasogastric suction loses hydrochloric acid will direct you to the option identifying an alkalotic condition. Because the question addresses a situation other than a respiratory one, the acid-base disorder would be a metabolic condition.

Which finding in a postoperative client would be of concern to the nurse? a. Temperature of 37.6°C (99.6°F) b. Blood pressure of 88/52 mm Hg c. Urinary output of 40 mL/hr d. Moderate drainage on the surgical dressing

ANS: B Rationale: The client's preoperative or baseline blood pressure is used to make informed postoperative comparisons. A falling systolic blood pressure, under 90 mm Hg, is considered reportable because it could be an indication of bleeding and potential shock. Urine output should be maintained at a minimum of 30 mL/hr for an adult, so 40 mL per hour is adequate. An output of less than 30 mL/hr for each of two consecutive hours should be reported to the health care provider. A temperature above 37.7°C (100°F) or below 36.1°C (97°F) is a concern and would be reportable. Moderate or light serous drainage from the surgical site is considered normal. Test-Taking Strategy: Focus on the subject, the assessment finding that is of concern to the nurse. To answer this question correctly, you must know the normal ranges for temperature, blood pressure, urinary output, and wound drainage. Then you can determine that the blood pressure is the only observation that is not within the normal range

The nurse has provided instructions for measuring blood glucose levels to a client newly diagnosed with diabetes mellitus who will be taking insulin. The client demonstrates understanding of the instructions by identifying which method as the best method for monitoring blood glucose levels? a. "I will check my blood glucose level every day at 5:00 p.m." b. "I will check my blood glucose level before each meal and at bedtime." c. "I will check my blood glucose level 2 hours after each meal." d. "I will check my blood glucose level 1 hour after each meal."

ANS: B Rationale: The most effective and accurate measure for testing blood glucose is to test the level before each meal and at bedtime. If possible and feasible, testing should be done during the nighttime hours. Checking the level after the meal will provide an inaccurate assessment of diabetes control. Checking the level once daily will not provide enough data to control the diabetes mellitus. Test-Taking Strategy: Note the strategic word, best. Eliminate options 2 and 3 first because they are comparable or alike. Next, eliminate option 1, knowing that once daily would not be an effective measure for monitoring diabetic control.

The nurse reviews the arterial blood gas results of a client and notes the following: pH 7.45, Paco2of 30 mm Hg (30 mm Hg), and HCO3- of 20 mEq/L (20 mmol/L). The nurse analyzes these results as indicating which condition? a. Metabolic acidosis, compensated b. Respiratory alkalosis, compensated c. Metabolic alkalosis, uncompensated d. Respiratory acidosis, uncompensated

ANS: B Rationale: The normal pH is 7.35 to 7.45. In a respiratory condition, an opposite effect will be seen between the pH and the Paco2. In this situation, the pH is at the high end of the normal value and the Pco2is low. In an alkalotic condition, the pH is elevated. Therefore, the values identified in the question indicate a respiratory alkalosis that is compensated by the kidneys through the renal excretion of bicarbonate. Because the pH has returned to a normal value, compensation has occurred. Test-Taking Strategy: Focus on the subject, arterial blood gas results. Remember that in a respiratory imbalance you will find an opposite response between the pH and the Pco2 as indicated in the question. Therefore, you can eliminate the options reflective of a primary metabolic problem. Also, remember that the pH increases in an alkalotic condition and compensation can be evidenced by a normal pH. The correct option reflects a respiratory alkalotic condition and compensation and describes the blood gas values as indicated in the question.

The nurse is preparing to care for a client with a potassium deficit. The nurse reviews the client's record and determines that the client is at risk for developing the potassium deficit because of which situation? a. Sustained tissue damage b. Requires nasogastric suction c. Has a history of Addison's disease d. Uric acid level of 9.4 mg/dL (high)

ANS: B Rationale: The normal serum potassium level is 3.5 to 5.0 mEq/L (3.5 to 5.0 mmol/L). A potassium deficit is known as hypokalemia. Potassium-rich gastrointestinal fluids are lost through gastrointestinal suction, placing the client at risk for hypokalemia. Other causes of hypokalemia include vomiting, diarrhea, or diuretic use. The client with tissue damage or Addison's disease and the client with hyperuricemia are at risk for hyperkalemia. The normal uric acid level for a female is 2.7 to 7.3 mg/dL (16 to 0.43 mmol/L) and for a male is 4.0 to 8.5 mg/dL (0.24 to 0.51 mmol/L). Hyperuricemia, often associated with Gout, is a cause of hyperkalemia.

The nurse has a prescription to hang a crystalloid intravenous solution of lactated Ringer's on a newly admitted client. The nurse notices that the client has a history of kidney disease. What action should the nurse take first? a. Hang the solution. b. Contact the health care provider (HCP). c. Check the client's daily laboratory results. d. Ask the client if any labs have ever been done to examine renal function.

ANS: B Rationale: The nurse must contact the HCP before administering the solution. Fluid and electrolyte replacement solutions like lactated Ringer's are contraindicated for clients with kidney and liver disease or lactic acidosis. Test-Taking Strategy: Focus on the subject, lactated Ringer's solution, and note the strategic word, first. Then note the data in the question, history of kidney disease

The nurse should include which interventions in the plan of care for a client with hyperthyroidism? Select all that apply. a. Instruct the client to consume a low-fat diet. b. Encourage the client to consume a well-balanced diet. c. A thyroid-releasing inhibitor will be prescribed. d. Provide a warm environment for the client. e. Instruct the client that episodes of chest pain are expected to occur. f. Instruct the client that thyroid replacement therapy will be needed.

ANS: B, C Rationale: The clinical manifestations of hyperthyroidism are the result of increased metabolism caused by high levels of thyroid hormone. Interventions are aimed at reduction of the hormones and measures to support the signs and symptoms related to an increased metabolism. The client often has heat intolerance and requires a cool environment. The nurse encourages the client to consume a well-balanced diet because clients with this condition experience increased appetite. Iodine preparations are used to treat hyperthyroidism. Iodine preparations decrease blood flow through the thyroid gland and reduce the production and release of thyroid hormone. Thyroid replacement is needed for hypothyroidism. The client would notify the health care provider if chest pain occurs because it could be an indication of an excessive medication dose. Test-Taking Strategy: Focus on the client's diagnosis and note the subject, hyperthyroidism. Recalling that in this disorder the client has an increased metabolic rate will assist you in determining the appropriate interventions.

The nurse teaches a client with diabetes mellitus about differentiating between hypoglycemia and ketoacidosis. The client demonstrates an understanding of the teaching by stating that a form of glucose should be taken if which symptom or symptoms develop? Select all that apply. a. Polyuria b. Lightheadedness c. Palpitations d. Shakiness e. Fruity Breath

ANS: B, C, D Rationale: Shakiness, palpitations, and lightheadedness are signs/symptoms of hypoglycemia and would indicate the need for food or glucose. Polyuria, blurred vision, and a fruity breath odor are manifestations of hyperglycemia.

The nurse is planning care for a client with deep vein thrombosis of the right leg. Which interventions would the nurse plan, based on the health care provider's (HCP's) prescriptions? Select all that apply. a. Ambulation in around the nursing unit every hour b. Monitoring for signs of pulmonary embolism c. Administration of acetaminophen d. Application of moist heat to the right leg e. Elevation of the right leg

ANS: B, C, D, E Rationale: Standard management of the client with deep vein thrombosis includes possible bed rest for 5 to 7 days or as prescribed; limb elevation; relief of discomfort with warm, moist heat and analgesics as needed; anticoagulant therapy; and monitoring for signs of pulmonary embolism. Although the health care provider may allow ambulation, hourly ambulation around the nursing unit is not encouraged because it increases the likelihood of dislodgement of the tail of the thrombus, which could travel to the lungs as a pulmonary embolism.

The nurse is monitoring a client who was diagnosed with type 1 diabetes mellitus and is being treated with NPH and regular insulin. Which manifestations would alert the nurse to the presence of a possible hypoglycemic reaction? Select all that apply. a. Hot, dry skin b. Irritability c. Nervousness d. Anorexia e. Tremors

ANS: B, C, E Rationale: Decreased blood glucose levels produce autonomic nervous system symptoms, which are manifested classically as nervousness, irritability, and tremors. Option 5 is more likely to occur with hyperglycemia. Options 2 and 6 are unrelated to the manifestations of hypoglycemia. In hypoglycemia, usually the client feels hunger.

The nurse is developing a plan of care for a client with Cushing's syndrome. The nurse documents a client problem of excess fluid volume. Which nursing actions should be included in the care plan for this client? Select all that apply. a. Maintain a low-potassium diet. b. Monitor daily weight. c. Assess extremities for edema. d. Maintain a high-sodium diet. e. Monitor intake and output.

ANS: B, C, E Rationale: The client with Cushing's syndrome and a problem of excess fluid volume should be on daily weights and intake and output and have extremities assessed for edema. He or she should be maintained on a high-potassium, low-sodium diet. Decreased sodium intake decreases renal retention of sodium and water. Test-Taking Strategy: Note the subject, Cushing's syndrome, and focus on the words excess fluid volume to assist in answering the question. Recall that a high sodium intake will contribute to the excess fluid volume. Also, recall that the client with Cushing's syndrome loses potassium. Options 1, 2, and 3 are appropriate interventions for the client with excess fluid volume.

The nurse is caring for a client with diabetic ketoacidosis and documents that the client is experiencing Kussmaul's respirations. Which patterns did the nurse observe? Select all that apply. a. Respirations that are shallow b. Respirations that are increased in rate c. Respirations that are abnormally slow d. Respirations that are abnormally deep

ANS: B, D Rationale: Kussmaul's respirations are abnormally deep and increased in rate to blow off extra CO2. These occur as a result of the compensatory action by the lungs. In bradypnea, respirations are regular but abnormally slow. Apnea is described as respirations that cease for several seconds. Test-Taking Strategy: Focus on the subject, the characteristics of Kussmaul's respirations. Use knowledge of the description of Kussmaul's respirations. Recalling that this type of respiration occurs in diabetic ketoacidosis to rid the body of excess CO2 (acid) will assist you in answering correctly.

The nurse is preparing a client for surgery scheduled in two hours. Which interventions are appropriate in the preoperative period? Select all that apply. a. Teach postoperative breathing exercises before the client is pre-medicated. b. Review the client's record for a history and physical report and laboratory reports. c. Administer all the daily medications 2 hours before the scheduled time of the surgery. d. Assist the client to void before transfer to the operating room. e. Check all surgeon's prescriptions to ensure they have been carried out.

ANS: B, D, E Rationale: The nurse should assist the client to void before transfer to the operating room, if a Foley catheter is not in place. The nurse also checks the surgeon's prescriptions to ensure that they have been carried out; if a prescription has not been carried out, the nurse would have the time to ensure that it is. Two hours before the scheduled surgery time is not the time to teach breathing exercises. This should have been accomplished earlier. A history and physical needs to be in the record so that all health care providers involved in the surgical procedure will be familiar with the client's health status. Additionally, the results of any laboratory tests prescribed need to be documented. The nurse does not administer all daily medications. Rather, the health care provider writes a specific prescription outlining which medications may be given with a sip of water. Test-Taking Strategy: Focus on the subject, the preoperative period. Recall that teaching should be done prior to the day of surgery; this will assist to eliminate option 3. Next eliminate option 5 because of the closed-ended word, all.

The nurse is performing an assessment on a client admitted to the hospital with a diagnosis of dehydration. Which assessment finding should the nurse expect to note? a. Bradycardia b. Elevated blood pressure c. Changes in mental status d. Bilateral crackles in the lungs

ANS: C Rationale: A client with dehydration is likely to be lethargic or complain of a headache. The client would also exhibit weight loss, sunken eyes, poor skin turgor, flat neck and peripheral veins, tachycardia as compensation to low blood pressure (recall baroreceptors). The client who is dehydrated would not have bilateral crackles in the lungs because these are signs of fluid overload and an unrelated finding of dehydration. Test-Taking Strategy: Focus on the subject, assessment findings in dehydration. Think about the pathophysiology that occurs in a dehydrated state when hypovolemia is present. This will direct you to the correct option.

The nurse is caring for four clients who will undergo surgery today. Which client does the nurse recognize as at highest risk for surgical complication? a. 69-year-old who will be discharged after surgery to an extended care facility b. 58-year-old who has well-controlled Type II diabetes c. 52-year-old who takes aspirin daily d. 64-year-old who has just received pre-surgical prophylactic antibiotics

ANS: C Rationale: Aspirin and NSAIDs taken before surgery may increase clotting time and risk for hemorrhage.

A client with a 3-day history of nausea and vomiting presents to the emergency department. The client is hypoventilating and has a respiratory rate of 10 breaths/minute. The electrocardiogram (ECG) monitor displays tachycardia, with a heart rate of 120 beats/minute. Arterial blood gases are drawn and the nurse reviews the results, expecting to note which finding? a. A decreased pH and a compensatory increased PaCO2 b. An increased pH and a compensatory decreased PaCO2 c. An increased pH and compensatory decreased HCO3- d. An increased pH and compensatory increase in HCO3

ANS: C Rationale: Clients experiencing nausea and vomiting would most likely present with metabolic alkalosis resulting from loss of gastric acid, thus causing the pH to increase and a compensatory decrease in the kidneys production of HCO3 to compensate. Symptoms experienced by the client would include hypoventilation (to hold on to Co2 and bring pH back down) and tachycardia. Option 1 reflects a respiratory acidotic condition. Option 2 reflects a respiratory alkalotic condition, and option 3 reflects a metabolic acidotic condition. Test-Taking Strategy: Focus on the subject, expected arterial blood gas findings. Note the data in the question and that the client is vomiting. Recalling that vomiting most likely causes metabolic alkalosis will assist in directing you to the correct option. Hyporventilation is an attempt to correct the alkaline pH, by holding on to more CO2.

A client is admitted to a hospital with a diagnosis of diabetic ketoacidosis (DKA). The initial blood glucose level is 950 mg/dL (54.2 mmol/L). A continuous intravenous (IV) infusion of short-acting insulin is initiated, along with IV rehydration with normal saline. The serum glucose level is now decreased to 240 mg/dL (13.7 mmol/L). The nurse would next prepare to administer which medication? a. NPH insulin subcutaneously b. An ampule of 50% dextrose c. IV fluids containing dextrose d. Phenytoin for the prevention of seizures

ANS: C Rationale: Emergency management of DKA focuses on correcting fluid and electrolyte imbalances and normalizing the serum glucose level. If the corrections occur too quickly, serious consequences, including hypoglycemia and cerebral edema, can occur. During management of DKA, when the blood glucose level falls to 250 to 300 mg/dL (14.2 to 17.1 mmol/L), the IV infusion rate is reduced and a dextrose solution is added to maintain a blood glucose level of about 250 mg/dL (14.2 mmol/L), or until the client recovers from ketosis. Fifty percent dextrose is used to treat hypoglycemia. NPH insulin is not used to treat DKA. Phenytoin is not a usual treatment measure for DKA. Test-Taking Strategy: Note the strategic word, next. Focus on the subject, management of DKA. Eliminate option 2 first, knowing that short-duration (rapid-acting) insulin is used in the management of DKA. Eliminate option 1 next, knowing that this is the treatment for hypoglycemia. Note the words the serum glucose level is now decreased to 240 mg/dL (13.7 mmol/L). This should indicate that the IV solution containing dextrose is the next step in the management of care.

What client teaching will the nurse provide regarding postoperative leg exercises, to minimize the risk for development of deep vein thrombosis after surgery? a. Only perform each exercise one time to prevent overuse. b. Bend knee, and push heel of foot into the bed until the calf and thigh muscles contract. Repeat several times, then switch legs. c. Point toes of one foot toward bottom of bed, then point toes of same leg toward their face. Repeat several times, then switch legs. d. Begin exercises by sitting at a 90-degree angle on the side of the bed.

ANS: C Rationale: Exercises should be repeated several times for each leg. Clients should begin by lying in the bed in a 45-degree angle. Pointing toes, as described, promotes circulation. Clients should push the ball of the foot into the bed until the calf and thigh muscles contract.

The nurse is caring for a client with a nasogastric (NG) tube who has a prescription for NG tube irrigation once every 8 hours. To maintain homeostasis, which solution should the nurse use to irrigate the NG tube? a. Tap Water b. Sterile Water c. 0.9% sodium chloride d. 0.45% sodium chloride

ANS: C Rationale: Homeostasis is maintained by irrigating with an isotonic solution, such as 0.9% sodium chloride. Tap water, sterile water, and 0.45% sodium chloride are hypotonic solutions. Test-Taking Strategy: Knowing that options that are comparable or alike are not likely to be correct, eliminate options 1 and 2 because they include water. Irrigating with solutions that contain water increases the risk for fluid and electrolyte imbalance. Select between the last two options knowing that an isotonic irrigating solution helps to maintain homeostasis and that 0.9% is isotonic; thus, option 4 can be eliminated.

The nurse is caring for a client after thyroidectomy. The nurse notes that calcium gluconate is prescribed for the client. The nurse determines that this medication has been prescribed for which purpose? a. To stimulate release of parathyroid hormone b. To treat thyroid storm c. To treat hypocalcemic tetany d. To prevent cardiac irritability

ANS: C Rationale: Hypocalcemia, resulting in tetany, can develop after thyroidectomy if the parathyroid glands are accidentally removed during surgery. Manifestations develop 1 to 7 days after surgery. If the client develops numbness and tingling around the mouth, fingertips, or toes; muscle spasms; or twitching, the health care provider is notified immediately. Calcium gluconate should be readily available in the nursing unit. Test-Taking Strategy: Focus on the subject, the purpose of calcium gluconate. Noting the name of the medication (calcium gluconate) should easily direct you to the correct option. Calcium would be given if hypocalcemia tetany occurs.

The nurse performs a physical assessment on a client with type 2 diabetes mellitus. Findings include a fasting blood glucose level of 120 mg/dL (6.8 mmol/L), temperature of 101°F (38.3°C), pulse of 102 beats/minute, respirations of 22 breaths/minute, and blood pressure of 142/72 mm Hg. Which finding would be the priority concern to the nurse? a. Pulse b. Respiration c. Temperature d. Blood Pressure

ANS: C Rationale: In the client with type 2 diabetes mellitus, an elevated temperature may indicate infection. Infection is a leading cause of hyperosmolar hyperglycemic syndrome in the client with type 2 diabetes mellitus. The other findings are within normal limits. Test-Taking Strategy: Note the strategic word, priority. Use knowledge of the normal values of vital signs to direct you to the correct option. The client's temperature is the only abnormal value. Remember that an elevated temperature can indicate an infectious process that can lead to complications in the client with diabetes mellitus.

The nurse is reading a health care provider's (HCP's) progress notes in the client's record and reads that the HCP has documented "insensible fluid loss of approximately 800 mL daily." The nurse makes a notation that insensible fluid loss occurs through which type of excretion? a. Urinary output b. Wound drainage c. Integumentary output d. The gastrointestinal tract

ANS: C Rationale: Insensible losses may occur without the person's awareness. Insensible losses occur daily through the skin and the lungs. Sensible losses are those of which the person is aware, such as through urination, wound drainage, and gastrointestinal tract losses. Test-Taking Strategy: Note that the subject of the question is insensible fluid loss. Note that urination, wound drainage, and gastrointestinal tract losses are comparable or alike in that they can be measured for accurate output. Fluid loss through the skin cannot be measured accurately; it can only be approximated.

A client is admitted to an emergency department, and a diagnosis of myxedema coma is made. Which action should the nurse prepare to carry out initially? a. Administer thyroid hormone b. Warm the client c. Maintain a patent airway d. Administer fluid replacement

ANS: C Rationale: Myxedema coma is a rare but serious disorder that results from persistently low thyroid production. Coma can be precipitated by acute illness, rapid withdrawal of thyroid medication, anesthesia and surgery, hypothermia, and the use of sedatives and opioid analgesics. In myxedema coma, the initial nursing action is to maintain a patent airway. Oxygen should be administered, followed by fluid replacement, keeping the client warm, monitoring vital signs, and administering thyroid hormones by the intravenous route. Test-Taking Strategy: Note the strategic word, initially. All the options are appropriate interventions, but use the ABCs-airway, breathing, and circulation-in selecting the correct option.

The nurse has instructed a preoperative client using an incentive spirometer to sustain the inhaled breath for 3 seconds. When the client asks about the rationale for this action, the nurse explains that this action achieves which function? a. Dilates the major bronchi b. Enhances ciliary action in the tracheobronchial tree c. Maintains inflation of the alveoli d. Increases surfactant production

ANS: C Rationale: Sustained inhalation helps maintain inflation of terminal bronchioles and alveoli, thereby promoting better gas exchange. Routine use of devices such as an incentive spirometer can help prevent atelectasis and pneumonia in clients at risk for these conditions. Options 1, 2, and 4 are incorrect. Test-Taking Strategy: Focus on the subject, client teaching about an incentive spirometer. Recall the anatomy and physiology of respiration to answer this question. Knowing that the alveoli constitute the most distal portion of the respiratory tree will help you to choose this as the area to derive the benefit from maximum sustained inhalation.

The nurse is caring for a client having respiratory distress related to an anxiety attack. Recent arterial blood gas values are pH = 7.53, Pao2 = 72 mm Hg (72 mm Hg), Paco2 = 32 mmHg (32 mm Hg), and HCO3- = 28 mEq/L (28 mmol/L). Which conclusion about the client should the nurse make? a. The client has acidotic blood. b. The client is fluid volume overloaded. c. The client is probably hyperventilating. d. The client has COPD

ANS: C Rationale: The ABG values are abnormal, which supports a physiological problem. The ABGs indicate respiratory alkalosis as a result of hyperventilating, not acidosis. No conclusion can be made about a client's fluid volume status from the information provided. COPD usually leads to respiratory acidosis due to CO2 retention. Test-Taking Strategy: Focus on the data in the question. Note the ABG values and use knowledge to interpret them. Note that the pH is elevated and the Paco2 is decreased from normal. This will assist you in determining that the client is experiencing respiratory alkalosis. Next, think about the causes of respiratory alkalosis to answer correctly.

The nurse is reviewing the laboratory test results for a client with a diagnosis of Cushing's syndrome. Which laboratory finding would the nurse expect to note in this client? a. A white blood cell (WBC) count of 6.0 b. A blood glucose level of 110 mg/d c. A potassium (K+) level of 3.0 mEq/L d. A platelet count of 200,000 mm3

ANS: C Rationale: The client with Cushing's syndrome experiences hypokalemia, hyperglycemia, an elevated WBC count, and elevated plasma cortisol and adrenocorticotropic hormone levels. These abnormalities are caused by the effects of excess glucocorticoids and mineralocorticoids in the body. The laboratory values listed in the remaining options would not be noted in the client with Cushing's syndrome. Test-Taking Strategy: Options 1, 2, and 4 are incorrect because they are within normal ranges and therefore are comparable or alike.

The PACU nurse caring for a client with a nasogastric (NG) tube notes 300 mL of bright red blood has collected. What is the appropriate nursing action? a. Immediately remove the NG tube. b. Place the client in Trendelenburg position. c. Call the client's surgeon to report the drainage. d. Document as a normal finding.

ANS: C Rationale: The presence of bright red blood reflects active bleeding. The nurse must report this to the surgeon immediately.

A client is recovering well 24 hours after cranial surgery but is fatigued. The surgeon advances the client from nothing-by-mouth (NPO) status to clear liquids. The nurse knows that which information is least reliable in determining the client's readiness to take in fluids? a. Presence of bowel sounds b. Absence of nausea c. Appetite d. Presence of a swallow reflex

ANS: C Rationale: To begin to tolerate oral intake after cranial or any other type of surgery, the client must have bowel sounds. The client also must have intact swallow and gag reflexes and should be free of nausea and vomiting. The client is likely to be easily fatigued, which may decrease appetite. Thus appetite is the least reliable indicator regarding when intake should be started. Test-Taking Strategy: Focus on the subject, the assessment item that is of little use in directing nursing actions related to diet. Think about each item identified in the options and its effect on readiness for eating to direct you to the correct choice.

When a client is transferred from the post-anesthesia care unit and arrives on the surgical unit, which should be the first action taken by the nurse? a. Check the rate of the intravenous infusion. b. Administer oxygen to the client. c. Obtain the client's vital signs. d. Assess the client's pain.

ANS: C Rationale: When a client arrives on the nursing unit from the post-anesthesia care unit (PACU), the nurse receives the client and immediately checks the client's airway status. The nurse next performs an initial assessment consisting of vital signs. The results must be compared with the vital signs last obtained in the PACU. Once this has been done, the intravenous infusion is checked, and a pain, respiratory, neurological, wound, urinary, and safety assessment is performed. Oxygen is not needed for every postoperative client but may be administered to those who may have a compromised respiratory status. A decision regarding how to intervene to control pain cannot be made until vital signs are known. The nurse documents the findings, including the time that the client arrived from the PACU. Test-Taking Strategy: Note the strategic word, first. Think about the priority actions. Eliminate options 3 and 4 because the health care provider's prescription would have to be checked first. Next, select option 2 because it relates to the ABCs-airway, breathing, and circulation.

The home health nurse visits a client with a diagnosis of type 1 diabetes mellitus. The client relates a history of vomiting and diarrhea and tells the nurse that no food has been consumed for the last 24 hours. Which additional statement by the client indicates a need for further teaching? a. "I need to call the health care provider (HCP) because of these symptoms." b. "I need to increase my fluid intake." c. "I need to stop my insulin." d. "I need to monitor my blood glucose every 3 to 4 hours."

ANS: C Rationale: When a client with diabetes mellitus is unable to eat normally because of illness, the client still should take the prescribed insulin or oral medication. The client should consume additional fluids and should notify the HCP. The client should monitor the blood glucose level every 3 to 4 hours. The client should also monitor the urine for ketones during illness. Test-Taking Strategy: Note the strategic words, need for further teaching. These words indicate a negative event query and the need to select the incorrect statement. Remembering that the type 1 diabetic client needs to take insulin will direct you easily to the correct option.

The nurse should include which interventions in the plan of care for a client with hypothyroidism? Select all that apply. a. Instruct the client to consume a high-fat diet. b. Provide a cool environment for the client. c. Instruct the client to contact the health care provider (HCP) if episodes of chest pain occur. d. Instruct the client about thyroid replacement therapy. e. Inform the client that iodine preparations will be prescribed to treat the disorder. f. Encourage the client to consume fluids and high-fiber foods in the diet.

ANS: C, D, F Rationale: The clinical manifestations of hypothyroidism are the result of decreased metabolism from low levels of thyroid hormone. Interventions are aimed at replacement of the hormone and providing measures to support the signs and symptoms related to decreased metabolism. The client often has cold intolerance and requires a warm environment. The nurse encourages the client to consume a well-balanced diet that is low in fat for weight reduction and high in fluids and high-fiber foods to prevent constipation. Iodine preparations may be used to treat hyperthyroidism. Iodine preparations decrease blood flow through the thyroid gland and reduce the production and release of thyroid hormone; they are not used to treat hypothyroidism. The client is instructed to notify the HCP if chest pain occurs because it could be an indication of overreplacement of thyroid hormone. Test-Taking Strategy: Focus on the subject, hypothyroidism. Recalling the manifestations of this disorder and that in this disorder the client has a decreased metabolic rate will assist in determining the appropriate interventions.

On review of the clients' medical records, the nurse determines that which client is at risk for fluid volume excess? a. The client taking diuretics and has tenting of the skin b. The client with an ileostomy from a recent abdominal surgery c. The client who requires intermittent gastrointestinal suctioning d. The client with kidney disease and a 12-year history of diabetes mellitus

ANS: D Rationale: A fluid volume excess is also known as overhydration or fluid overload and occurs when fluid intake or fluid retention exceeds the fluid needs of the body. The causes of fluid volume excess include decreased kidney function, heart failure, use of hypotonic fluids to replace isotonic fluid losses, excessive irrigation of wounds and body cavities, and excessive ingestion of sodium. The client taking diuretics, the client with an ileostomy, and the client who requires gastrointestinal suctioning are at risk for fluid volume deficit. Test-Taking Strategy: Focus on the subject, fluid volume excess. Think about the pathophysiology associated with fluid volume excess. Read each option and think about the fluid imbalance that can occur in each. Clients taking diuretics or having ileostomies or gastrointestinal suctioning all lose fluid. The only condition that can cause an excess is the condition noted in the correct option.

The nurse is performing an assessment on a client with a diagnosis of left-sided heart failure. Which assessment component would elicit specific information regarding the client's left-sided heart function? a. Assessing for peripheral and sacral edema b. Listening to lung sounds c. Palpating for organomegaly (organ enlargement) d. Assessing for jugular vein distention

B. Listening to lung sounds Rationale: The client with heart failure may present with different symptoms, depending on whether the right or the left side of the heart is failing. Peripheral and sacral edema, jugular vein distention, and organomegaly all are manifestations of problems with right-sided heart function. Lung sounds constitute an accurate indicator of left-sided heart function. Test-Taking Strategy: Focus on the subject, left-sided heart failure. Correlate left and lungs. Options 2, 3, and 4 reflect right-sided heart failure.

The nurse is obtaining a pulse oximetry reading from a postoperative client who appears short of breath. The client has dark fingernail polish on top of artificial nails. What is the most appropriate action? a. Obtain fingernail polish remover, remove the polish, and then obtain the pulse oximetry reading from a finger. b. Check labs, Hgb and Hct. c. Take the pulse oximetry reading from any finger. d. Obtain a pulse oximetry reading from another appropriate area, such as an earlobe. e. Remove one of the artificial nails and then obtain the reading from the finger.

ANS: D Rationale: A pulse oximetry reading may not provide an accurate measurement if it is measured on a finger that has dark polish and an artificial nail; therefore, option 1 is not the most appropriate action. It is not appropriate to remove an artificial nail; therefore, eliminate option 2. Removing the polish and taking the reading with the artificial nail may provide a better reading than taking the reading with the polish; however, this is not the most appropriate action from those provided, so option 4 can be eliminated. Although Hgb and Hct provide insight into the patient's oxygen carrying capacity, you would not forgo the pulse ox check as an initial priority assessment. Test-Taking Strategy: Note the strategic words, most appropriate. Focus on the subject, obtaining a pulse oximetry reading. Think about this procedure. Recalling that there is more than one anatomical area to take a reading will direct you to option 3.

In preparation for ambulation, the nurse is planning to assist a postoperative client to progress from a lying position to a sitting position. Which nursing action is appropriate to maintain the safety of the client? a. Elevate the head of the bed quickly to assist the client to a sitting position. b. Assist the client to move quickly from the lying position to the sitting position. c. Allow the client to rise from the bed to a standing position unassisted. d. Assess the client for signs of dizziness and hypotension.

ANS: D Rationale: Early ambulation should not exceed the client's tolerance. The client should be assessed before sitting. The client is assisted to rise from the lying position to the sitting position gradually until any evidence of dizziness, if present, has subsided. This position can be achieved by raising the head of the bed slowly. After sitting, the client may be assisted to a standing position. The nurse should be at the client's side to provide physical support and encouragement. Test-Taking Strategy: Eliminate options 3 and 4 because of the word quickly and option 2 because of the word unassisted. Additionally, the correct option is the only option that reflects assessment, the first step of the nursing process.

A client with a gastric ulcer is scheduled for surgery. The client cannot sign the operative consent form because of sedation from opioid analgesics that have been administered. The nurse should take which most appropriate action in the care of this client? a. Obtain a court order for the surgery. b. Have the charge nurse sign the informed consent immediately. c. Send the client to surgery without the consent form being signed. d. Obtain a telephone consent from a family member, following agency policy.

ANS: D Rationale: Every effort should be made to obtain permission from a responsible family member to perform surgery if the client is unable to sign the consent form. A telephone consent must be witnessed by 2 persons who hear the family member's oral consent. The 2 witnesses then sign the consent with the name of the family member, noting that an oral consent was obtained. Consent is not informed if it is obtained from a client who is confused, unconscious, mentally incompetent, or under the influence of sedatives. In an emergency, a client may be unable to sign and family members may not be available. In this situation, a health care provider is permitted legally to perform surgery without consent, but the data in the question do not indicate an emergency. Options 1, 2, and 3 are not appropriate in this situation. Also, agency policies regarding informed consent should always be followed.

The nurse is conducting preoperative teaching with a client about the use of an incentive spirometer. The nurse should include which piece of information in discussions with the client? a. Inhale as rapidly as possible. b. Keep a loose seal between the lips and the mouthpiece. c. After maximum inspiration, hold the breath for 15 seconds and exhale. d. The best results are achieved when sitting up or with the head of the bed elevated 45 to 90 degrees.

ANS: D Rationale: For optimal lung expansion with the incentive spirometer, the client should assume the semi Fowler's or high Fowler's position. The mouthpiece should be covered completely and tightly while the client inhales slowly, with a constant flow through the unit. The breath should be held for 5 seconds before exhaling slowly.

The nurse has delegated taking orthostatic vital signs to the unlicensed assistive personnel (UAP). The UAP reports the following vital signs. Which client should the nurse assess as the priority? a. Lying BP: 118/76 mmHg; standing BP 128/88 mmHg b. Lying BP: 144/94 mm Hg; standing BP 136/88 mmHg c. Lying BP: 122/86 mmHg; standing BP 116/78 mmHg d. Lying BP: 136/96 mmHg; standing BP 134/76 mmHg

ANS: D Rationale: Orthostatic hypotension is demonstrated with a 20-point drop in systolic pressure when the client moves from lying to sitting to standing or a 10-point drop in diastolic pressure. Although 134/76 mmHg is not abnormally low, it does demonstrate a large diastolic drop, a positive sign for orthostatic hypotension. None of the other readings satisfy these criteria.

A client who is found unresponsive has arterial blood gases drawn and the results indicate the following: pH is 7.12, Paco2 is 90 mm Hg (90 mm Hg), and HCO3- is 22 mEq/L (22 mmol/L). The nurse interprets the results as indicating which condition? a. Metabolic acidosis with compensation b. Respiratory acidosis with compensation c. Metabolic acidosis without compensation d. Respiratory acidosis without compensation

ANS: D Rationale: The acid-base disturbance is respiratory acidosis without compensation. The normal pH is 7.35 to 7.45. The normal Paco2 is 35 to 45 mm). In respiratory acidosis the pH is decreased and the Pco2 is elevated. The normal bicarbonate (HCO3-) level is 21 to 28 mEq/L (21 to 28 mmol/L). Because the bicarbonate is still within normal limits, the kidneys have not had time to adjust for this acid-base disturbance. In addition, the pH is not within normal limits. Therefore, the condition is without compensation. The remaining options are incorrect interpretations. Test-Taking Strategy: Focus on the subject, interpretation of arterial blood gas results. Remember that in a respiratory imbalance you will find an opposite response between the pH and the Paco2. Also, remember that the pH is decreased in an acidotic condition and that compensation is reflected by a normal pH.

A nurse assesses a patient with tachycardia. Which clinical manifestation requires immediate intervention by the nurse? a. Mild orthostatic hypotension b. Midsternal chest pain c. Increased urine output d. P wave touching the T wave

B. Midsternal chest pain

The nurse is preparing a preoperative client for transfer to the operating room. The nurse should take which action in the care of this client at this time? a. Administer all the daily medications. b. Verify that the client has not eaten for the past 24 hours. c. Have the client practice postoperative breathing exercises. d. Ensure that the client has voided.

ANS: D Rationale: The nurse should ensure that the client has voided if a Foley catheter is not in place. The nurse does not administer all daily medications just before sending a client to the operating room. Rather, the health care provider writes a specific prescription outlining which medications may be given with a sip of water. The time of transfer to the operating room is not the time to practice breathing exercises; this should have been done earlier. The client has nothing by mouth for 6 to 8 hours before surgery, not 24 hours. Test-Taking Strategy: Focus on the subject, action to take just before transfer to the operating room. This tells you that you must prioritize your answer according to a time line. With this in mind, eliminate options 2 and 3. Choose correctly between the remaining options by knowing that the client must empty his or her bladder or by knowing that the client is likely to be anxious at this time, making it inappropriate to practice breathing exercises.

A client has returned to the nursing unit after an abdominal hysterectomy. The client is lying supine. To thoroughly assess the client for postoperative bleeding, what is the primary nursing action? a. Check the blood pressure. b. Ask the client about sensation of moistness on her perineal pad. c. Check the heart rate. d. Roll the client to one side and check her perineal pad.

ANS: D Rationale: The nurse should roll the client to one side after checking the perineal pad for vaginal bleeding and the abdominal dressing. This client position allows the nurse to check the rectal area, where blood may pool by gravity if the client is lying supine. Asking the client about a sensation of moistness is not a complete assessment. Vital signs will change with hemorrhage; they are a compensatory mechanism of change. Assess for external or most likely signs of bleeding first. Test-Taking Strategy: Note the strategic word, primary. Focus on the subject, how to thoroughly assess the client for postoperative bleeding after an abdominal hysterectomy. Eliminate option 4 first because it relies on the client. Regarding the remaining options, note that the correct option addresses rolling the client, which would provide a thorough assessment. Thorough assessment also includes always visually assessing the surgical site.

The nurse is creating a plan of care for a client scheduled for surgery. The nurse should include which activity in the nursing care plan for the client on the day of surgery? a. Verify that the client has not eaten for the last 24 hours. b. Report immediately any slight increase in blood pressure or pulse. c. Avoid oral hygiene and rinsing with mouthwash. d. Have the client void immediately before going into surgery.

ANS: D Rationale: The nurse would assist the client to void immediately before surgery so that the bladder will be empty. Oral hygiene is allowed, but the client should not swallow any water. The client usually has a restriction of food and fluids for 6 to 8 hours (or longer as prescribed) before surgery instead of 24 hours. A slight increase in blood pressure and pulse is common during the preoperative period and is usually the result of anxiety.

The nurse has just reassessed the condition of a postoperative client who was admitted 1 hour ago to the surgical unit. The nurse plans to monitor which parameter most carefully during the next hour? a. Temperature of 37.6°C (99.6°F) b. Serous drainage on the surgical dressing c. Blood pressure of 100/70 mm Hg d. Urinary output of 20 mL/hour

ANS: D Rationale: Urine output should be maintained at a minimum of 30 mL/hour for an adult. An output of less than 30 mL for 2 consecutive hours should be reported to the health care provider. A temperature higher than 37.7°C (100°F) or lower than 36.1°C (97°F) and a falling systolic blood pressure, lower than 90 mm Hg, are usually considered reportable immediately. The client's preoperative or baseline blood pressure is used to make informed postoperative comparisons. Moderate or light serous drainage from the surgical site is considered normal.

The nurse is providing preoperative teaching to a client scheduled for a cholecystectomy (gall bladder removal). Which intervention would be of highest priority in the preoperative teaching plan? a. Teaching leg exercises b. Providing instructions regarding fluid restrictions c. Assessing the client's understanding of the surgical procedure d. Teaching coughing and deep breathing exercises

ANS: D Rationale: Although all answer choices are important, after cholecystectomy, respirations tend to be shallow because deep breathing is painful as a result of the location for the surgical procedure. Although all the options are correct, teaching coughing and deep breathing exercises is the highest priority. Test-Taking Strategy: Note the strategic words, highest priority. Use the ABCs-airway, breathing, and circulation-to answer the question. The correct option relates to airway. Additionally, recalling the anatomical location for the surgical procedure will assist in directing you to the correct option.

When performing a surgical dressing change on a client's abdominal dressing, the nurse notes an increased amount of drainage and separation of the incision line. The underlying tissue is visible to the nurse. The nurse should take which action in the initial care of this wound? a. Apply a sterile dressing soaked in povidone-iodine. b. Irrigate the wound and apply a sterile dry dressing. c. Leave the incision open to the air to dry the area. d. Apply a sterile dressing soaked with normal saline.

ANS: D Rationale: Wound dehiscence is the separation of wound edges at the suture line. Signs and symptoms include increased drainage and the visible appearance of underlying tissues. Dehiscence usually occurs 6 to 8 days after surgery. The client should be instructed to remain quiet and to avoid coughing or straining. The client should be positioned to prevent further stress on the wound (semi Fowler's position). Sterile dressings soaked with sterile normal saline should be used to cover the wound. The nurse must notify the health care provider after applying this initial dressing to the wound. Options 1, 2, and 4 are incorrect. Test-Taking Strategy: Focus on the subject, wound dehiscence. Note the strategic word, initial. Eliminate option 1 first because this action would dry the wound and would also present a risk of infection to the underlying tissues. Eliminate options 2 and 4 next because a dry dressing or a dressing soaked with povidone-iodine would irritate the exposed body tissues.

A nurse cares for a patient with diabetes mellitus who asks, "Why do I need to administer more than one injection of insulin each day?" How would the nurse respond? a. "A single dose of insulin would be too large to be absorbed, predictably putting you at risk for insulin shock." b. "A single dose of insulin each day would not match your blood insulin levels and your food intake patterns." c. "A regimen of a single dose of insulin injected each day would require that you eat fewer carbohydrates." d. "You need to start with multiple injections until you become more proficient at self-injection."

B. "A single dose of insulin each day would not match your blood insulin levels and your food intake patterns."

After teaching a patient who is recovering from a complete thyroidectomy, the nurse assesses the patient's understanding. Which statement made by the patient indicates a need for additional instruction? a. "I can receive pain medication if I feel that I need it." b. "After surgery, I won't need to take thyroid medication." c. "I'll need to take thyroid hormones for the rest of my life." d. "I may need calcium replacement after surgery."

B. "After surgery, I won't need to take thyroid medication."

A nurse cares for a patient who has a heart rate averaging 56 beats/min with no adverse symptoms. Which activity modification would the nurse suggest to avoid further slowing of the heart rate? a. "Make certain that your bath water is warm." b. "Avoid straining while having a bowel movement." c. "Avoid strenuous exercise such as running." d. "Limit your intake of caffeinated drinks to one a day."

B. "Avoid straining while having a bowel movement."

The nurse educator is teaching the new registered nurse (RN) how to care for clients with a decrease in blood pressure. Which statement by the new RN indicates the need for further instruction? a. "Decreased contractility occurs." b. "Decreased myocardial blood flow is not a concern." c. "Decreased heart rate is not a side effect." d. "Increased resistance to electrical stimulation often occurs."

B. "Decreased myocardial blood flow is not a concern." Rationale: The primary effect of a decrease in blood pressure is reduced blood flow to the myocardium. This in turn decreases oxygenation of the cardiac tissue. Cardiac tissue is likely to become more excitable or irritable in the presence of hypoxia. Correspondingly, the heart rate is likely to increase, not decrease, in response to this change. The effects of tissue ischemia lead to decreased contractility over time.

The nurse assesses a patient with chronic obstructive pulmonary disease (COPD). Which questions would the nurse ask to determine the patient's activity tolerance? (Select all that apply) a. "Have you lost any weight lately?" b. "How long does it take to perform your morning routine?" c. "Do you walk upstairs every day?" d. "Do you have any difficulty sleeping?"

B. "How long does it take to perform your morning routine?" C. "Do you walk upstairs every day?"

A patient has peripheral arterial disease (PAD). What statement by the patient indicates misunderstanding about self-management activities? a. "I should not cross my legs when sitting or lying down." b. "I can use a heating pad on my legs if it's set on low." c. "I will go out and buy some warm, heavy socks to wear." d. "It's going to be really hard but I will stop smoking."

B. "I can use a heating pad on my legs if it's set on low."

A client is admitted to the visiting nurse service for assessment and follow-up after being discharged from the hospital with new-onset heart failure (HF). The nurse teaches the client about the dietary restrictions required with HF. Which statement by the client indicates that further teaching is needed? a. "I can have most fresh fruits and vegetables." b. "I'm going to have a ham and cheese sandwich and potato chips for lunch." c. "I'm going to weigh myself daily to be sure I don't gain too much fluid." d. "I'm not supposed to eat cold cuts."

B. "I'm going to have a ham and cheese sandwich and potato chips for lunch." Rationale: When a client has HF, the goal is to reduce fluid accumulation. One way that this is accomplished is through sodium reduction. Ham (and most cold cuts), cheese, and potato chips are high in sodium. Daily weighing is an appropriate intervention to help the client monitor fluid overload. Most fresh fruits and vegetables are low in sodium. Test-Taking Strategy: Note the strategic words, further teaching is needed. This phrasing indicates a negative event query and asks you to select an incorrect statement. Recalling that the goal for clients with HF is to reduce fluid accumulation will direct you to the correct option.

The nurse educator is lecturing new registered nurses (RNs) about serum calcium levels. Which statement by one of the new RNs indicates that teaching has been effective? a. "Low calcium levels cause high blood pressure." b. "Low calcium levels can lead to cardiac arrest." c. "Calcium has no effect on the risk for stroke." d. "Calcium has no effect on urinary stone formation."

B. "Low calcium levels can lead to cardiac arrest." Rationale: The normal calcium level is 9 to 10.5 mg/dL (2.25 to 2.75 mmol/L). A low calcium level could lead to severe ventricular dysrhythmias, prolonged QT interval, and ultimately cardiac arrest. Calcium is needed by the heart for contraction. Calcium ions move across cell membranes into cardiac cells during depolarization and move back during repolarization. Depolarization is responsible for cardiac contraction. Options 1 and 3 are unrelated to calcium levels. Elevated calcium levels can lead to urinary stone formation. The nurse would take action and contact the health care provider when a calcium level is abnormal. Test-Taking Strategy: Note the strategic word, effective. Eliminate options 1 and 3 because they are unrelated to calcium levels. Next recall the effects of calcium on the cardiac system and that urinary stone formation could result from hypercalcemia. This will assist in answering correctly.

A nurse is caring for a patient who was prescribed high-dose corticosteroid therapy for 1 month to treat a severe inflammatory condition. The patient's symptoms have now resolved and the patient asks, "When can I stop taking these medications?" How would the nurse respond? a. "The drug suppresses your immune system, which must be built back up." b. "Once you start corticosteroids, you have to be weaned off them." c. "It is possible for the inflammation to recur if you stop the medication." d. "You must decrease the dose slowly so your hormones will work again."

B. "Once you start corticosteroids, you have to be weaned off them."

The new registered nurse (RN) is reviewing cardiac rhythms with a mentor. Which statement by the new RN indicates that teaching about ventricular fibrillation has been effective? a. "Ventricular fibrillation has recognizable P waves, QRS complexes, and T waves." b. "Ventricular fibrillation does not have P waves or QRS complexes." c. "Ventricular fibrillation is a regular pattern of wide QRS complexes." d. "Ventricular fibrillation appears as irregular beats within a rhythm."

B. "Ventricular fibrillation does not have P waves or QRS complexes." Rationale: Ventricular fibrillation is characterized by the absence of P waves and QRS complexes. The rhythm is instantly recognizable by the presence of coarse or fine fibrillatory waves on the cardiac monitoring screen. Premature ventricular contractions (PVCs) appear as irregular beats within a rhythm. Ventricular tachycardia is a regular pattern of wide QRS complexes. Sinus tachycardia has a recognizable P wave, QRS complex, and T wave. Each of the incorrect options has a recognizable complex that appears on the monitoring screen. Test-Taking Strategy: Note the strategic word, effective, and focus on the subject, the characteristics of ventricular fibrillation. Note the description of the pattern in each option. Thinking about the word fibrillation and about what fibrillation may produce will direct you to the correct option. Remember that no true complexes are present in ventricular fibrillation.

A registered nurse (RN) is providing instructions to an unlicensed assistive personnel (UAP) assigned to give a bed bath to a client who is on contact precautions. The RN instructs the UAP to use which protective item when giving the bed bath? a. Gloves and shoe protectors b. A gown and gloves c. A gown and goggles d. Gloves and goggles

B. A gown and gloves Rationale: Contact precautions require the use of gloves and a gown if direct client contact is anticipated. Goggles are not necessary unless it is anticipated that splashes of blood, bodily fluids, secretions, or excretions may occur. Shoe protectors are not necessary. Test-Taking Strategy: Focus on the subject, protective items needed for contact precautions. Eliminate option a first, knowing that shoe protectors are not necessary. The question contains no information indicating that splashes will occur; therefore, eliminate options c and d.

Your patient has arthritis that affects the weight-bearing joints such as the hands, knees, hips, and spine. This type of arthritis is most likely: a. Rheumatoid arthritis b. Osteoarthritis

B. Osteoarthritis Rationale: Osteoarthritis is a form of arthritis that causes deterioration of the articular hyaline cartilage of the bones. It affects the weight-bearing joints. This can include the hands, knees, hips, and spine because these joints experience a lot of stress.

The nurse is caring for a client hospitalized with acute exacerbation of chronic obstructive pulmonary disease. Which findings would the nurse expect to note on assessment of this client? Select all that apply. a. A widened diaphragm noted on the chest x-ray b. A hyperinflated chest noted on the chest x-ray c. Decreased oxygen saturation with mild exercise d. Pulmonary function tests that demonstrate increased vital capacity e. A low arterial PCo2 level

B. A hyperinflated chest noted on the chest x-ray C. Decreased oxygen saturation with mild exercise Rationale: Clinical manifestations of chronic obstructive pulmonary disease (COPD) include hypoxemia, hypercapnia, dyspnea on exertion and at rest, oxygen desaturation with exercise, and the use of accessory muscles of respiration. Chest x-rays reveal a hyperinflated chest and a flattened diaphragm if the disease is advanced. Pulmonary function tests will demonstrate decreased vital capacity. Test-Taking Strategy: Focus on the subject, manifestations of COPD. Think about the pathophysiology associated with this disorder. Remember that hypercapnia, a hyperinflated chest, a flat diaphragm, oxygen desaturation on exercise, and decreased vital capacity are manifestations.

A client with chronic obstructive pulmonary disease (COPD) who is beginning oxygen therapy asks the nurse how to manage the amount of oxygen given. How should the nurse instruct the client? a. Adjust the oxygen depending on respiratory rate. b. Adjust the oxygen depending on SpO2. c. Do not exceed 2 L/min. d. Do not exceed 1 L/min.

B. Adjust the oxygen depending on SpO2 Rationale: The client with COPD is often dependent on oxygen, and has compensated for chronically lower O2 levels and higher CO2 levels. The oxygen should be adjusted depending on the SpO2, which should be 88% to 92%. Increasing the O2 too much, resulting too high of SpO2 should be avoided in COPD patients. All other options are incorrect. Test-Taking Strategy: Focus on the subject, oxygen delivery rate for the client with COPD. Use knowledge of basic respiratory physiology and note that the SpO2 is acceptable between 88% and 92% with this disorder.

At 4:45 PM, a nurse assesses a patient with diabetes mellitus who is recovering from an abdominal hysterectomy 2 days ago. The nurse notes that the patient is confused and diaphoretic. The nurse reviews the assessment data provided in the chart below: Capillary Blood Glucose Testing (AC/HS) Dietary Intake At 06:30—95At 11:30—70At 16:30—47 Breakfast: 10% eaten—patient states that she is not hungry Lunch: 5% eaten—patient is nauseous; vomits once After reviewing the patient's assessment data, which action is appropriate at this time? a. Assess the patient's oxygen saturation level and administer oxygen. b. Administer dextrose 50% intravenously and reassess the patient. c. Provide a glass of orange juice and encourage the patient to eat dinner. d. Reorient the patient and apply a cool washcloth to the patient's forehead.

B. Administer dextrose 50% intravenously

The nurse assesses a patient who reports waking up feeling very tired, even after 9 hours of undisturbed sleep. What action would the nurse take first? a. Request a prescription for a sleep aid from the prescriber b. Ask the patient if he or she has ever been evaluated for sleep apnea. c. Advise the patient to avoid beverages with caffeine before bed. d. Educate the patient regarding benefits of sleeping upright in a reclining chair.

B. Ask the patient if he or she has ever been evaluated for sleep apnea.

A client with a complete heart block has had a permanent demand ventricular pacemaker inserted. The nurse assesses for proper pacemaker function by examining the electrocardiogram (ECG) strip for the presence of pacemaker spikes at what point? a. Just after each P wave b. Before each QRS complex c. During each P wave d. Just after each T wave

B. Before each QRS complex Rationale: If a ventricular pacemaker is functioning properly, there will be a pacer spike followed by a QRS complex. An atrial pacemaker spike precedes a P wave if an atrial pacemaker is implanted. Test-Taking Strategy: Focus on the subject, proper pacemaker function. Note the word ventricular, and use knowledge of the normal cardiac conduction pathway as represented on the ECG tracing to answer this question. Knowing that the QRS complex signals ventricular depolarization will direct you to the correct option.

The nurse is caring for a patient on the medical-surgical unit who suddenly becomes unresponsive and has no pulse. The cardiac monitor shows the rhythm. After calling for assistance and a defibrillator, what action would the nurse take next? a. Ask the patient's family about code status. b. Initiate cardiopulmonary resuscitation (CPR). c. Start an 18-gauge intravenous line. d. Perform a pericardial thump.

B. Initiate cardiopulmonary resuscitation (CPR)

An external insulin pump is prescribed for a client with diabetes mellitus. When the client asks the nurse about the functioning of the pump, the nurse bases the response on which information about the pump? a. It is surgically attached to the pancreas and infuses regular insulin into the pancreas. This releases insulin into the bloodstream. b. It administers a small continuous dose of short-duration insulin subcutaneously. The client can self-administer an additional bolus dose from the pump before each meal. c. It is timed to release programmed doses of either short-duration or NPH insulin into the bloodstream at specific intervals. d. It continuously infuses small amounts of NPH insulin into the bloodstream while regularly monitoring blood glucose levels.

B. It administers a small continuous dose of short-duration insulin subcutaneously. The client can self-administer an additional bolus dose from the pump before each meal. Rationale: An insulin pump provides a small continuous dose of short-duration (rapid- or short-acting) insulin subcutaneously throughout the day and night. The client can self-administer an additional bolus dose from the pump before each meal as needed. Short-duration insulin is used in an insulin pump. An external pump is not attached surgically to the pancreas. Test-Taking Strategy: Focus on the subject, use of an insulin pump. Recalling that short-duration insulin is used in an insulin pump will assist in eliminating options 1 and 2. Noting the word external in the question will assist in eliminating option 3.

A nurse cares for a patient who presents with bradycardia secondary to hypothyroidism. Which medication does the nurse prepare to administer? a. Atropine sulfate b. Levothyroxine sodium (Synthroid) c. Propranolol (Inderal) d. Epinephrine (Adrenalin)

B. Levothyroxine sodium (Synthroid)

The nurse instructs a client to use the pursed-lip method of breathing and evaluates the teaching by asking the client about the purpose of this type of breathing. The nurse determines that the client understands if the client states that the primary purpose of pursed-lip breathing is to promote which outcome? a. Strengthen the intercostal muscles. b. Promote carbon dioxide elimination. c. Strengthen the diaphragm. d. Promote oxygen intake.

B. Promote carbon dioxide elimination Rationale: Pursed-lip breathing facilitates maximal expiration for clients with obstructive lung disease. This type of breathing allows better expiration by increasing airway pressure that keeps air passages open during exhalation. Options 1, 2, and 3 are not the purposes of this type of breathing.

The nurse is assisting in planning care for a client with a diagnosis of immunodeficiency and should incorporate which action as a priority in the plan? a. Identifying factors that decreased the immune function b. Protecting the client from infection c. Encouraging discussion about lifestyle changes d. Providing emotional support to decrease fear

B. Protecting the client from infection Rationale: The client with immunodeficiency has inadequate or absence of immune bodies and is at risk for infection. The priority nursing intervention would be to protect the client from infection. Options a, c, and d may be components of care but are not the priority. Test-Taking Strategy: Note the strategic word, priority. Use Maslow's Hierarchy of Needs theory to answer the question. Remember that physiological needs are the priority. This will direct you to the correct option.

An arterial blood gas report indicates the client's pH is 7.30, PCO2 is 55 mm Hg and HCO3 is 20 mEq/L. These results are consistent with: a. Metabolic acidosis b. Respiratory acidosis c. Metabolic alkalosis d. Respiratory alkalosis

B. Respiratory acidosis

A client recovering from an exacerbation of left-sided heart failure is experiencing activity intolerance. Which change in vital signs during activity would be the best indicator that the client is tolerating mild exercise? a. Blood pressure decreased from 140/86 to 112/72 mm Hg. b. Respiratory rate increased from 16 to 19 breaths per minute. c. Oxygen saturation decreased from 96% to 91%. d. Pulse rate increased from 80 to 104 beats per minute.

B. Respiratory rate increased from 16 to 19 breaths per minute. Rationale: Vital signs that remain near baseline indicate good cardiac reserve with exercise. Only the respiratory rate remains within the normal range. In addition, it reflects a minimal increase. A pulse rate increase to a rate more than 100 beats per minute during mild exercise does not show tolerance, nor does a 5% decrease in oxygen saturation levels. In addition, blood pressure decreasing by more than 10 mm Hg is not a sign indicating tolerance of activity. Test-Taking Strategy: Note the strategic word, best, and focus on the subject, the indicator that the client is tolerating exercise. The only option that identifies values that remain within the normal range is the correct one.

A nurse cares for a patient who possibly has syndrome of inappropriate antidiuretic hormone (SIADH). The patient's serum sodium level is 114 mEq/L (114 mmol/L). What action would the nurse take first? a. Consult with the dietitian about increased dietary sodium b. Restrict the patient's fluid intake to 600 mL/day. c. Handle the patient gently by using turn sheets for repositioning. d. Instruct unlicensed assistive personnel to measure intake and output.

B. Restrict the patient's fluid intake to 600 mL/day

A nurse reviews the laboratory results of a patient who is receiving intravenous insulin. Which would alert the nurse to intervene immediately? a. Serum sodium level of 132 mEq (132 mmol/L) b. Serum potassium level of 2.5 mEq/L (2.5 mmol/L) c. Serum calcium level of 8.8 mg/dL (2.2 mmol/L) d. Serum chloride level of 98 mEq/L (98 mmol/L)

B. Serum potassium level of 2.5 mEq/L (2.5 mmol/L)

A nurse evaluates laboratory results for a patient with heart failure. Which results would the nurse expect? (Select all that apply.) a. Serum potassium: 4.0 mEq/L (4.0 mmol/L) b. Serum sodium: 130 mEq/L (130 mmol/L) c. Serum creatinine: 1.0 mg/dL (88.4 mcmol/L) d. Proteinuria e. Hematocrit: 32.8%

B. Serum sodium: 130 mEq/L D. Proteinuria E. Hematocrit: 32.8%

The nurse is instructing a hospitalized client with a diagnosis of emphysema about measures that will enhance the effectiveness of breathing during dyspneic periods. Which position should the nurse instruct the client to assume? a. Sitting up in bed b. Sitting up and leaning on an overbed table c. Side-lying in bed d. Sitting in a recliner chair

B. Sitting up and leaning on an overbed table Rationale: Positions that will assist the client with emphysema with breathing include sitting up and leaning on an overbed table, sitting up and resting the elbows on the knees, and standing and leaning against the wall.

A client has just returned from the cardiac catheterization laboratory. The left-sided femoral vessel was used as the access site. How should the nurse position the client? Select one: a. Supine, with the head of the bed elevated 45 to 90 degrees b. Supine, with the head of the bed elevated about 15 degrees c. Knee chest, with the foot of the bed elevated d. Semi Fowler's, with the knees placed on top of 1 pillow

B. Supine, with the head of the bed elevated about 15 degrees Rationale: Following cardiac catheterization, the extremity used for catheter insertion is kept straight for 4 to 6 hours. If the femoral artery was used, strict bed rest is necessary for 6 to 12 hours. The client may turn from side to side. The head of the bed is not elevated more than 15 degrees (unless otherwise prescribed) to prevent kinking of the blood vessel at the groin and possible arterial occlusion.

A client who is experiencing respiratory difficulty asks the nurse, "Why it is so much easier to breathe out than in?" In providing a response, the nurse explains that breathing is easier on exhalation because of which respiratory responses? a. Air is flowing against a pressure gradient. b. The respiratory muscles relax. c. The respiratory muscles contract. d. Air flows by gravity.

B. The respiratory muscles relax Rationale: Exhalation is less taxing for the client because it is a passive process in which the respiratory muscles relax. This allows air to flow upward out of the lungs. Air flows according to a pressure gradient from higher pressure to lower pressure. It does not flow by gravity or against a pressure gradient. Test-Taking Strategy: Knowledge that respiratory muscles tighten or contract with inspiration and relax with expiration will lead you to the correct option.

While assessing a patient with Graves' disease, the nurse notes that the patient's temperature has risen 1° F (1° C). What does the nurse do first? a. Calculate the patient's apical-radial pulse deficit. b. Turn the lights down and shut the patient's door. c. Call for an immediate electrocardiogram (ECG). d. Administer a dose of acetaminophen (Tylenol).

B. Turn the lights down and shut the patient's door

An oxygen delivery system is prescribed for a client with chronic obstructive pulmonary disease to deliver a precise oxygen concentration. Which oxygen delivery system would the nurse prepare for the client? a. Aerosol mask b. Venturi mask c. Face tent d. Tracheostomy collar

B. Venturi mask Rationale: The Venturi mask delivers the most accurate oxygen concentration. It is the best oxygen delivery system for the client with chronic airflow limitation such as chronic obstructive pulmonary disease, because it delivers a precise oxygen concentration. The face tent, aerosol mask, and tracheostomy collar are also high-flow oxygen delivery systems but most often are used to administer high humidity.

The nurse suspects that a patient is deficient in thyroid-stimulating hormone. What assessment findings would correlate to this condition? (Select all that apply.) a. Hyperactivity b. Weight gain c. Alopecia d. Decreased libido

B. Weight gain C. Alopecia D. Decreased libido

The nurse is discharging a client with chronic obstructive pulmonary disease (COPD) and reviewing specific instructional points about COPD. What comment by the client indicates that further teaching is needed? a. "I need to avoid alcohol and sedative medications." b. "Besides smoking, I can't be around second- or thirdhand smoke." c. "I have to keep my nasal cannula oxygen levels between 4 and 6 L/minute." d. "I have to cut down on the percentage of carbohydrates in my diet."

C. "I have to keep my nasal cannula oxygen levels between 4 and 6 L/minute." Rationale: Clients with COPD have adapted to a high carbon dioxide level, so their carbon dioxide-sensitive chemoreceptors are essentially not functioning. Their stimulus to breathe is a decreased arterial oxygen (PaO2) level, so administration of oxygen greater than 24% to 28% (1 to 3 L/min) prevents the PaO2 from falling to a level (60 mm Hg) that stimulates the peripheral receptors, thus destroying the stimulus to breathe. The resulting hypoventilation causes excessive retention of carbon dioxide, which can lead to respiratory acidosis and respiratory arrest. Therefore, oxygen administration levels for clients with COPD should be kept within the range of 1 to 3 L/min (per health care provider prescription). Also, nutrition for the client with COPD requires a reduction in the percentage of carbohydrates in the diet. Excessive carbohydrate loads increase carbon dioxide production, which the client with COPD may be unable to exhale. In addition to avoiding alcohol and sedative medications, the increased risk for COPD from active smoking, passive smoking (or secondhand smoke), and smoke that clings to hair and clothing (sometimes called "thirdhand" smoke), contributes to upper and lower respiratory problems. Test-Taking Strategy: Note the strategic words, further teaching is needed. These words indicate a negative event query and ask you to select a statement that is inaccurate. Recall the pathophysiology of breathing for clients with COPD. With these clients, the administration of excessive oxygen results in hypoventilation.

A nurse assesses a patient with diabetes mellitus and notes that the patient only responds to a sternal rub by moaning, has capillary blood glucose of 33 mg/dL, and has an intravenous line that is infiltrated with 0.45% normal saline. What action would the nurse take first? a. Encourage the patient to drink orange juice. b. Administer 25 mL dextrose 50% (D50) IV push. c. Administer 1 mg of intramuscular glucagon. d. Insert a new intravenous access line.

C. Administer 1 mg of intramuscular glucagon

A nurse assesses a patient who has diabetes mellitus and notes that the patient is awake and alert, but shaky, diaphoretic, and weak. Five minutes after administering a half-cup (120 mL) of orange juice, the patient's clinical manifestations have not changed. What action would the nurse take next? a. Administer 1 mg of glucagon intramuscularly. b. Administer a half-ampule of dextrose 50% intravenously. c. Administer another half-cup (120 mL) of orange juice d. Administer 10 units of regular insulin subcutaneously.

C. Administer another half-cup (120 mL) of orange juice

A nurse is interested in providing community education and screening on hypertension. In order to reach a priority population, to what target audience would the nurse provide this service? a. Asian-American groceries b. Women's health clinics c. African-American churches d. High school sports camps

C. African-American churches

An erythrocyte sedimentation rate (ESR) determination is prescribed for a client with a connective tissue disorder. The client asks the nurse about the purpose of the test. What should the nurse tell the client about the purpose of the test? a. Confirms the diagnosis of a connective tissue disorder b. Determines the presence of antigens c. Confirms the presence of inflammation or infection in the body d. Identifies which additional tests need to be performed

C. Confirms the presence of inflammation or infection in the body Rationale: The ESR is a blood test that can confirm the presence of inflammation or infection in the body. It is particularly useful for the management of connective tissue disease because the rate measured directly correlates with the degree of inflammation and later with the severity of the disease. The other options are incorrect. Test-Taking Strategy: Focus on the subject, the purpose of an ESR determination. Knowledge of the purpose of the ESR and a focus on the client's diagnosis will assist you in answering this question. Remember that the ESR can confirm inflammation or infection.

You are providing discharge teaching to a patient with newly diagnosed asthma. What would be the priority goal to established as part of the teaching plan prior to being discharged from the hospital? The patient will: a. Is able to obtain pulse oximeter readings b. Has identified a pharmacy for prescription medications c. Demonstrates correct use of prescribed inhalers d. Knows the primary care provider's office hours

C. Demonstrates correct use of prescribed inhalers

The nurse is performing a health screening on a 54-year-old client. The client has a blood pressure of 118/78 mm Hg, total cholesterol level of 190 mg/dL (4.9 mmol/L), and fasting blood glucose level of 184 mg/dL (10.2 mmol/L). The nurse interprets this to mean that the client has which modifiable risk factor for coronary artery disease (CAD)? a. Hyperlipidemia b. Hypertension c. Glucose Intolerance d. Age

C. Glucose intolerance Rationale: Hypertension, cigarette smoking, and hyperlipidemia are modifiable risk factors that are predictors of CAD. Glucose intolerance, obesity, and response to stress are contributing modifiable risk factors for CAD. Age is a nonmodifiable risk factor. The nurse places priority on risk factors that can be modified. In this scenario, the abnormal value is the fasting blood glucose level, indicating glucose intolerance as the priority risk factor. Test-Taking Strategy: Note the subject, modifiable risk factors for CAD. Options 2 and 3 can be eliminated first because the client's blood pressure and cholesterol level are within normal ranges. From the remaining options, eliminate option 1 because it is a nonmodifiable risk factor.

The nurse is caring for a client with cardiac disease who has been placed on a cardiac monitor. The nurse notes that the client has developed atrial fibrillation and has a rapid ventricular rate of 150 beats/minute. The nurse should next assess the client for which finding? a. Flat neck veins. b. Complaints of headache. c. Hypotension. d. Complaints of nausea.

C. Hypotension Rationale: The client with uncontrolled atrial fibrillation with a ventricular rate greater than 100 beats/minute is at risk for low cardiac output because of loss of atrial kick. The nurse assesses the client for palpitations, chest pain or discomfort, hypotension, pulse deficit, fatigue, weakness, dizziness, syncope, shortness of breath, and distended neck veins. Test-Taking Strategy: Note the strategic word, next. Focus on the dual subjects, atrial fibrillation and a rapid ventricular rate. Eliminate option 2 first because flat neck veins are normal or indicate hypovolemia. Eliminate option 3 next because nausea and vomiting would be associated with vagus nerve activity, which does not correlate with a tachycardic state. Regarding the remaining choices, recall that a falling cardiac output will result in hypotension.

The nurse working in a long-term care facility is assessing a client who is experiencing chest pain. The nurse should interpret that the pain is most likely caused by myocardial infarction (MI) on the basis of what assessment finding? a. The client is not experiencing nausea or vomiting. b. The client is not experiencing dyspnea. c. The pain has not been relieved by rest and nitroglycerin tablets. d. The client says the pain began while she was trying to open a stuck dresser drawer.

C. The pain has not been relieved by rest and nitroglycerin tablets. Rationale: The pain of MI is not relieved by rest and nitroglycerin and requires opioid analgesics, such as morphine sulfate, for relief. The pain of angina may radiate to the left shoulder, arm, neck, or jaw. It often is precipitated by exertion or stress, is accompanied by few associated symptoms, and is relieved by rest and nitroglycerin. The pain of MI also may radiate to the left arm, shoulder, jaw, and neck. It typically begins spontaneously, lasts longer than 30 minutes, and frequently is accompanied by associated symptoms (such as nausea, vomiting, dyspnea, diaphoresis, or anxiety). Test-Taking Strategy: Note the strategic words, most likely. Seek the option that differentiates anginal pain from that of MI. Recalling that a classic hallmark of the pain from MI is that it is unrelieved by rest and nitroglycerin will direct you to the correct option.

The nurse is caring for a client with emphysema who is receiving oxygen. The nurse assesses the oxygen flow rate and notes that the client is receiving 2 L/min. The client's SpO2 level is 86%. Based on this assessment, which action is appropriate? a. Place the client on a nonrebreather mask on 100% FiO2. b. Maintain at 2 L/min and call respiratory therapy for a breathing treatment. c. Increase to 3 L/min and titrate until the SpO2 is 92%. d. Increase to 3 L/min and titrate until the SpO2 is 95%.

C. Increase to 3 L/min and titrate until the SpO2 is 92% Rationale: Oxygen is used cautiously and should be titrated to the lowest amount needed. The current recommendation is that hypoxia should be treated with oxygen and that oxygen should be titrated to keep the SpO2 level between 88% and 92%. An SpO2 of 95% is the recommended level for a healthy individual and may not be achieved in a patient with long-standing emphysema; therefore, option 1 is incorrect. A nonrebreather mask is not necessary at this point, and increasing oxygen via nasal cannula should be attempted first; therefore, option 3 is incorrect. It may be necessary to call respiratory therapy for a breathing treatment; however, the oxygen needs to be titrated, making option 4 incorrect. Test-Taking Strategy: Focus on the subject, oxygen therapy for a client who has emphysema and the potential for hypoxia. Note the word appropriate. Note that there are two parts to each option and that both parts need to be correct. Recall that in the client with emphysema, and all other respiratory conditions, hypoxia needs to be treated. Therefore option 4 can be deleted. Focus on the data in the question to eliminate 3 because a nonrebreather is unnecessary. For the remaining options, focus on the client's diagnosis and recall that a SpO2 of 95% is the recommended level for a healthy individual.

A patient is in anaphylactic shock. The patient has a severe allergy to peanuts and mistakenly consumed an eggroll containing peanut ingredients during his lunch break. The patient is given Epinephrine intramuscularly. As the nurse, you know this medication will have what effect on the body? a. It will cause vasoconstriction and decrease the blood pressure. b. It will prevent a recurrent attack. c. It will help dilate the airways. d. It will help block the effects of histamine in the body.

C. It will help dilate the airways. Rationale: Epinephrine acts as a vasopressor and will actually dilate the airway. Epinephrine performs vasoconstriction which will INCREASE the blood pressure. It does not prevent a recurrent attack (corticosteroids may help with this), and it does not block the effects of histamine (antihistamine helps with this).

A nurse assesses a patient with Cushing's disease. Which assessment findings would the nurse correlate with this disorder? (Select all that apply.) a. Weight loss b. Hypotension c. Muscle atrophy d. Petechiae e. Moon face

C. Muscle atrophy D. Petechiae E. Moon face

The nurse is assisting a health care provider with the removal of a chest tube. The nurse should instruct the client to take which action to lessen chance of repeat pneumothorax? a. Exhale very quickly. b. Inhale and exhale quickly. c. Perform the Valsalva maneuver. d. Stay very still.

C. Perform the Valsalva maneuver Rationale: When the chest tube is removed, the client is asked to perform the Valsalva maneuver (take a deep breath, exhale, and bear down). The tube is quickly withdrawn, and an airtight dressing is taped in place. An alternative instruction is to ask the client to take a deep breath and hold the breath while the tube is removed.

Two nurses are leaving the room of a client whose care required them to wear a gown, mask, and gloves. Which action by these nurses could lead to the spread of infection? a. Taking off the gloves first before removing the gown b. Washing the hands after the entire procedure has been completed c. Removing the gown without rolling it from inside out d. Removing the gloves and then removing the gown using the neck ties

C. Removing the gown without rolling it from inside out. Rationale: The gown must be rolled from inside out to prevent the organisms on the outside of the gown from contaminating other areas. Gloves are considered the dirtiest piece of equipment and therefore must be removed first. Hands must be washed after removal of the protective garb to remove any unwanted germs still present. Ungloved hands should be used to remove the gown to prevent contaminating the back of the gown with germs from the gloves. Test-Taking Strategy: Focus on the subject, an incorrect action, and use knowledge regarding standard precautions to assist in answering the question. Visualize the process of removing protective garb, remembering to remove the dirtiest items first. Be sure to understand the order in which to remove protective garb. A slight change in the order can have devastating effects and compromise the care of the client.

The physician orders a patient in septic shock to receive a large IV fluid bolus. How would the nurse know if this treatment was successful for this patient? a. Patient's skin is warm and flushed. b. Patient's CVP 2 mmHg c. The patient's blood pressure changes from 75/48 to 110/82. d. Patient's urinary output is 20 mL/hr.

C. The patient's blood pressure changes from 75/48 to 110/82. Rationale: In septic shock, the first treatment is to try to maintain tissue perfusion with fluids. If that doesn't work to increase the blood pressure and maintain perfusion, vasopressors will be used next. In septic shock, the intravascular space will be depleted of fluid due to an increase in capillary permeability. This will lead to hypovolemia, which will decrease blood pressure and lead to a decrease in blood flow to organs/tissue. If the blood pressure increases to a normal state, that tells us the fluids are working. Adequate urine output is 30mL/hr or greater.

A nurse assesses a patient's electrocardiograph tracing and observes that not all QRS complexes are preceded by a P wave. How would the nurse interpret this observation? a. The patient's chest leads are not making sufficient contact with the skin. b. The patient has hyperkalemia causing irregular QRS complexes. c. Ventricular and atrial depolarizations are initiated from different sites. d. Ventricular tachycardia is overriding the normal atrial rhythm.

C. Ventricular and atrial depolarizations are initiated from different sites.

A nurse evaluates prescriptions for a patient with chronic atrial fibrillation. Which medication would the nurse expect to find on this patient's medication administration record to prevent a common complication of this condition? a. Lidocaine (Xylocaine) b. Sotalol (Betapace) c. Warfarin (Coumadin) d. Atropine (Sal-Tropine)

C. Warfarin (Coumadin)

A client recovering from pulmonary edema is preparing for discharge. What should the nurse plan to teach the client to do to manage or prevent recurrent symptoms after discharge? a. Withhold prescribed digoxin if slight respiratory distress occurs. b. Take a double dose of the diuretic if peripheral edema is noted. c. Weigh self on a daily basis. d. Sleep with the head of the bed flat.

C. Weigh self on a daily basis Rationale: The client can best determine fluid status at home by weighing himself or herself on a daily basis. Increases of 2 to 3 lb (0.9 to 1.4 kg) in a short period are reported to the health care provider (HCP). The client should sleep with the head of the bed elevated. During recumbent sleep, fluid (which has seeped into the interstitium with the assistance of the effects of gravity) is rapidly reabsorbed into the systemic circulation. Sleeping with the head of the bed flat is therefore avoided. The client does not modify medication dosages without consulting the HCP. Test-Taking Strategy: Eliminate options 3 and 4 first because they are comparable or alike; also, it is unsafe for clients to regulate their own medication dosages based on symptoms. Knowing that weight is an excellent indicator of fluid volume status will allow you to choose correctly between the remaining options.

The home health nurse makes a home visit to a client who has an implanted cardioverter-defibrillator (ICD) and reviews the instructions concerning pacemakers and dysrhythmias with the client. Which client statement indicates that further teaching is necessary? a. "My wife knows how to call the emergency medical services (EMS) if I need it." b. "If I feel an internal defibrillator shock, I should sit down." c. "I won't be able to have a magnetic resonance imaging test (MRI)." d. "I can stop taking my antidysrhythmic medicine now because I have a pacemaker."

D. "I can stop taking my antidysrhythmic medicine now because I have a pacemaker." Rationale: Clients with an ICD usually continue to receive antidysrhythmic medications after discharge from the hospital. The nurse should stress the importance of continuing to take these medications as prescribed. The nurse should provide clear instructions about the purposes of the medications, dosage schedule, and side effects or adverse effects to report. Clients should sit down if they feel an internal defibrillator shock. They cannot have an MRI because of the possible magnetic properties of the device. Also, knowledge of how to reach EMS is important.

The nurse is assisting in administering immunizations as well as providing education to the clients who receive them at a health care clinic. Which statement by a client indicates that teaching was successful? a. "Immunizations can provide innate immunity." b. "Immunizations can provide natural immunity." c. "Immunizations protect against all diseases." d. "Immunizations are a way to acquire immunity to a specific disease."

D. "Immunizations are a way to acquire immunity to a specific disease." Rationale: Acquired immunity is immunity that can occur by receiving an immunization that causes antibodies to a specific pathogen to form. No immunization protects the client from all diseases. Natural (innate) immunity is present at birth.

A nurse assesses a patient admitted to the cardiac unit. Which statement by the patient alerts the nurse to the possibility of right-sided heart failure? a. "I wake up coughing every night." b. "I have trouble catching my breath." c. "I sleep with four pillows at night." d. "My shoes fit really tight lately."

D. "My shoes fit really tight lately."

The nurse provides education to the client about the primary purpose of neutrophils. Which statement by the client indicates successful teaching? a. "They close up blood vessels." b. "They increase fluids at the injury site." c. "They open up blood vessels." d. "They engulf any potential foreign materials."

D. "They engulf any potential foreign materials." Rationale: Neutrophil function provides protection after invaders, especially bacteria, enter the body. In the inflammatory response, neutrophils appear in the area of injury in 30 to 60 minutes. Their primary purpose is to phagocytize (ingest and destroy) any potentially harmful agents, such as microorganisms. The remaining options are incorrect.

The nursing student conducted a clinical conference on the role of B lymphocytes in the immune system. Which statement by a fellow nursing student indicates successful teaching? a. "They activate T cells." b. "They attack and kill the target cell directly." c. "They initiate phagocytosis." d. "They produce antibodies."

D. "They produce antibodies." Rationale: B lymphocytes have the job of making antibodies and mediating humoral immunity. They do not activate T cells. T cells attack and kill target cells directly. The primary function of macrophages is phagocytosis. Test-Taking Strategy: Focus on the subject, the role of B lymphocytes. Knowledge of the function of B lymphocytes is required to answer this question. Remember that B lymphocytes have the job of making antibodies and mediating humoral immunity.

A nurse assesses a patient who is scheduled for a cardiac catheterization. Which assessment would the nurse complete prior to this procedure? a. Ability to turn self in bed b. Patient's level of anxiety c. Cardiac rhythm and heart rate d. Allergies to iodine-based agents

D. Allergies to iodine-based agents

A client is returned to the nursing unit after thoracic surgery with chest tubes in place. During the first few hours postoperatively, what type of drainage should the nurse expect? a. Serous b. Serosanguineous c. Bloody, with frequent small clots d. Bloody

D. Bloody Rationale: In the first few hours after surgery, the drainage from the chest tube is bloody. After several hours, it becomes serosanguineous. The client should not experience frequent clotting. Proper chest tube function should allow for drainage of blood before it has the chance to clot in the chest or the tubing. Test-Taking Strategy: Focus on the subject, expected findings after thoracic surgery. Recall that after thoracic surgery, there may be considerable capillary oozing for hours in the postoperative period. This will lead you to choose the bloody drainage option over the serous or serosanguineous drainage options. Knowing that patent chest tubes do not allow blood to collect in the pleural space eliminates the option of blood with clots.

A client is scheduled for a cardiac catheterization to diagnose the extent of coronary artery disease. The nurse places highest priority on telling the client to report which sensation during the procedure? a. Urge to cough b. Pressure at insertion site c. Warm, flushed feeling d. Chest pain

D. Chest pain Rationale: The client is taught to report chest pain or any unusual sensations immediately. The client also is told that he or she may be asked to cough or breathe deeply from time to time during the procedure. The client is informed that a warm, flushed feeling may accompany dye injection and is normal. Because a local anesthetic is used, the client is expected to feel pressure at the insertion site. Test-Taking Strategy: Note the strategic words, highest priority. Noting the relationship between the client's diagnosis and the correct option will direct you to answer correctly.

A nurse assesses a patient who is recovering from a total thyroidectomy and notes the development of stridor. What action does the nurse take first? a. Place the patient in high-Fowler's position and apply oxygen b. Document the finding and assess the patient hourly. c. Reassure the patient that the voice change is temporary. d. Contact the provider and prepare for intubation.

D. Contact the provider and prepare for intubation

A client has experienced an episode of pulmonary edema. The nurse determines that the client's respiratory status is improving after this episode if which breath sounds are noted? a. Wheezes b. Crackles throughout the lung fields c. Rhonchi d. Crackles in the bases

D. Crackles in the bases Rationale: Pulmonary edema is characterized by extreme breathlessness, dyspnea, air hunger, and the production of frothy, pink-tinged sputum. As the client's condition improves, the amount of fluid in the alveoli decreases, which may be detected by crackles in the bases. (Clear lung sounds indicate full resolution of the episode.) Rhonchi and wheezes are not associated with pulmonary edema. Auscultation of the lungs reveals crackles throughout the lung fields when pulmonary edema is at its worst. Test-Taking Strategy: Note the subject, breath sounds associated with an improvement in pulmonary edema. Fluid in the lungs from pulmonary edema produces sounds that are called crackles, which eliminates options 1 and 2. From the remaining options, eliminate option 4, noting the words respiratory status is improving in the question. Crackles throughout the lung fields do not indicate an improvement in the client's condition, but as pulmonary edema resolves the crackles reduce to the bases before disappearing all together.

A patient had a percutaneous transluminal coronary angioplasty for peripheral arterial disease. What assessment finding by the nurse indicates that a priority outcome for this patient has been met? a. Pain rated as 2/10 after medication b. Remains on bedrest as directed c. Verbalizes understanding of procedure d. Distal pulse on affected extremity 2+/4+

D. Distal pulse on affected extremity 2+/4+

A client with chronic obstructive pulmonary disease (COPD) has a respiratory rate of 24 breaths per minute, bilateral crackles, and cyanosis and is coughing but unable to expectorate sputum. Which problem is the priority? a. Gas exchange alteration related to ventilation-perfusion mismatch b. Altered breathing pattern secondary to increased work of breathing c. Low cardiac output secondary to cor pulmonale (right-sided heart failure) d. Inability to clear the airway related to inability to expectorate sputum

D. Inability to clear the airway related to inability to expectorate sputum Rationale: COPD is a term that represents the pathology and symptoms that occur with clients experiencing both emphysema and chronic bronchitis. All of the problems listed are potentially appropriate for a client with COPD. For the nurse prioritizing this client's problems, it is important first to maintain circulation, airway, and breathing. At present, the client demonstrates problems with ventilation because of ineffective coughing, so the correct option would be the priority problem. The bilateral crackles would suggest fluid or sputum in the alveoli or airways; however, the client is unable to expectorate this sputum. The client's respiratory rate is only slightly elevated, so option 3, altered breathing pattern, is not as important as airway. The client is cyanotic, but this probably is because of the ineffective clearance of the sputum, causing poor gas exchange. The data in the question do not support low cardiac output as being most important at this time. Test-Taking Strategy: Note the strategic word, priority. Focus on the data in the question, and note that the client is unable to expectorate sputum. Use the ABCs-airway, breathing, and circulation. This will direct you to the correct option.

A hospitalized client has been diagnosed with heart failure as a complication of hypertension. In explaining the disease process to the client, the nurse identifies which chamber of the heart as primarily responsible for the symptoms? a. Right atrium b. Left atrium c. Right ventricle d. Left ventricle

D. Left ventricle Rationale: Hypertension increases the workload of the left ventricle because the ventricle has to pump the stroke volume against increased resistance (afterload) in the major blood vessels. Over time this causes the left ventricle to fail, leading to signs and symptoms of heart failure. The remaining options are not the chambers that are primarily responsible for this disease process, although these chambers may be affected as the disease becomes more chronic.

The nurse in the medical unit is reviewing the laboratory test results for a client who has been transferred from the intensive care unit (ICU). The nurse notes that a cardiac troponin T assay was performed while the client was in the ICU. The nurse determines that this test was performed to assist in diagnosing which condition? a. Heart failure. b. Atrial fibrillation. c. Ventricular tachycardia. d. Myocardial infarction.

D. Myocardial infarction Rationale: Cardiac troponin T or cardiac troponin I have been found to be a protein marker in the detection of myocardial infarction, and assay for this protein is used in some institutions to aid in the diagnosis of a myocardial infarction. The test is not used to diagnose heart failure, ventricular tachycardia, or atrial fibrillation. Test-Taking Strategy: Focus on the subject, cardiac troponin levels. Specific knowledge of the cardiac troponin test is needed to answer this question. Think about each condition identified in the options and the method of diagnosing the condition to direct you to the correct option.

A nurse is caring for a patient with a deep vein thrombosis (DVT). What nursing assessment indicates that a priority outcome has been met? a. Verbalizing risk factors b. Pain of 2/10 after medication c. Ambulates with assistance d. Oxygen saturation of 98%

D. Oxygen saturation of 98%

The nurse is caring for a client with a wound infected with methicillin-resistant Staphylococcus aureus (MRSA). The most appropriate infection control precautions for MRSA include which intervention? a. Private room with negative-pressure airflow b. Room with positive-pressure airflow c. Mask or respiratory protection device and gown d. Private room, gown, gloves, and face shield

D. Private room, gown, gloves, and face shield Rationale: Isolation guidelines from the Centers for Disease Control and Prevention (CDC) place MRSA at the tier 2 transmission category. Contact precautions are required and include a private room, gloves, gowns, and face shields in case a splash from the wound drainage occurs, such as with wound irrigation. A room with negative-pressure airflow is required for airborne precautions from small droplet infections such as measles, chickenpox, or tuberculosis. A respiratory protection device is recommended for larger droplet infections such as pneumonia. A room with positive-pressure airflow is recommended for protective environments such as those required for clients with stem cell transplants. Test-Taking Strategy: Note the strategic words, most appropriate, and focus on the subject, MRSA precautions. Consider safety and disease transmission. Noting that the client has a wound that is infected will direct you to the correct option.

Which statement is FALSE concerning rheumatoid arthritis? a. Rheumatoid arthritis can occur at any age (20-60 year old most commonly). b. Ankylosis can occur in severe cases of rheumatoid arthritis. c. Rheumatoid arthritis most commonly affects the fingers and wrist. d. Rheumatoid arthritis is different from osteoarthritis in that it doesn't affect other systems of the body.

D. Rheumatoid arthritis is different from osteoarthritis in that it doesn't affect other systems of the body. Rationale: This statement is false. It should say that, "Rheumatoid arthritis is different from osteoarthritis in that it DOES (not doesn't) affect other systems of the body. RA is systemic, while OA only affects the joints. This is why a fever and anemia can present in RA. Ankylosis is a stiffness of a joint due to abnormal adhesion and rigidity of the bones of the joint, which may be the result of injury or disease. It is present in both osteoarthritis and rheumatoid arthritis, but more severe and longer lasting in rheumatoid.

The nurse is reinforcing instructions to a hospitalized client with heart block about the fundamental concepts regarding the cardiac rhythm. The nurse explains to the client that the normal site in the heart responsible for initiating electrical impulses is which site? a. Atriventricluar (AV) node b. Bundle of His c. Purkinje fibers d. Sinoatrial (SA) node

D. Sinoatrial (SA) node Rationale: The SA node is responsible for initiating electrical impulses that are conducted through the heart. The impulse leaves the SA node and travels down through internodal and interatrial pathways to the AV node. From there, impulses travel through the bundle of His to the right and left bundle branches and then to the Purkinje fibers. This group of specialized cardiac cells is referred to as the cardiac conduction system. The ability of this specialized tissue to generate its own impulses is called automaticity. Test-Taking Strategy: Focus on the subject, the area of the heart responsible for electrical impulses. Note the words initiating electrical impulses. Use knowledge of anatomy and physiology. Recalling the normal pathways of the cardiac conduction system will direct you to the correct option.

The home health nurse is watching the caregiver change the sternotomy dressing on the postoperative client. Which action by the caregiver identifies correct principles of infection control? a. The caregiver dons gloves before removal of the old dressing and then applies the new dressing. b. The caregiver selects a previously opened gauze to cover the sternal wound. c. The caregiver covers her mouth with her hand when she sneezes and then continues with the dressing change. d. The caregiver washes her hands before removal of the soiled dressing and again before applying the clean dressing.

D. The caregiver washes her hands before removal of the soiled dressing and again before applying the clean dressing. Rationale: The single most effective technique to prevent the spread of germs and bacteria is hand washing. The initial step with all aseptic procedures is hand washing. Using previously opened gauze, not washing the hands after sneezing, and not applying new gloves after removing the old dressing increase the risk of wound contamination as a result of poor aseptic technique. Test-Taking Strategy: Focus on the subject, principles of infection control. Gauze that has been opened has been exposed to potential pathogens. In option 2, the caregiver contaminates the gloves by removing the old dressing and then contaminates the clean dressing with the contaminated gloves. In option 3, the caregiver sneezes into her gloved hand and does not change the glove, thereby contaminating the dressing.

The nursing instructor asks a nursing student to identify the components of natural resistance as it relates to the immune system. All of the following are characteristics of natural resistance EXCEPT? a. "It includes all antigen-specific immunities a person develops during a lifetime." b. "It is the immunity with which a person is born." c. "It does not require previous exposure to the antigen." d. "It also is called inherited immunity."

a. "It includes all antigen-specific immunities a person develops during a lifetime." Rationale: Natural resistance, also called innate inherited or innate-native immunity, is the immunity with which a person is born. It does not require previous exposure to the antigen. Acquired immunity includes all antigen-specific immunities that a person develops during a lifetime.


Related study sets

Chapter 11: high-risk perinatal care- existing conditions

View Set

Chapter 34 Patients with Special Challenges

View Set

Ms. Szymanski Physics Chapter 21

View Set

Chapter 35: Assessment of Immune Function

View Set

MOTIVATIONAL NEEDS AND PROCESSES

View Set